health assessment 1

Pataasin ang iyong marka sa homework at exams ngayon gamit ang Quizwiz!

Madeline Leininger identified attributes and behaviors that a nurse practicing effective care across cultures must have. What statement is reflective of Leininger's theory? A. The nurse's own beliefs always interfere with cultural sensitive care. B. Use of active listening by the nurse is fundamental to effective care. C. Awareness of meanings behind the client's social communications is of minor importance. D. Use of body language is not recommended when providing cultural sensitive nursing care.

ANS: B Feedback: Leininger suggests that the attributes and behaviors of a nurse practicing effective care within the client's cultural context include genuine interest in a client's culture and personal life experiences, active listening, and awareness of meanings behind the client's verbal communication (storytelling); nonverbal communication (body language, eye contact, facial expressions, interpersonal space, and preferences regarding touch); and acknowledgement that the nurse's own beliefs and prejudices might create barriers to providing culturally sensitive care.

What do nursing activities that promote health and prevent disease accomplish? (Select all that apply.) A. Reduce the risk of disease B. Maintain optimal functioning C. Reinforce good habits D. Optimize self-care abilities E. Create home care safety

A., B., C. Feedback: Nursing activities that promote health and prevent illness reduce the risk of disease, reinforce good habits, and maintain optimal functioning. They do not optimize self-care abilities or create home care safety.

An older adult client hospitalized with pancreatitis is having problems performing ADLs. The nurse's aide brushes the client's hair and tells the nurse that the client is losing an excessive amount of hair. The hair has the hair bulb intact. What might this indicate to the nurse? A. Abnormal endocrine function B. Abnormal ovarian function C. Abnormal hepatic function D. Abnormal integumentary function

ANS: A Feedback: Absent hair bulb may indicate chemical damage to the hair shaft (excessive coloration). Presence of hair bulb may indicate abnormal endocrine function. There is no indication of abnormal ovarian, hepatic, or integumentary function with the loss of hair with the hair bulb intact.

A nursing instructor is discussing mental health assessment with a class of nursing students. While reviewing risk factors for mental illness, what would the instructor be sure to identify as a factor that cannot be changed? A. Age B. Occupation C. Environment D. Support systems

ANS: A Feedback: Factors that cannot be changed include family history, age, and gender. Occupation, environment, and support systems are all, to some degree, modifiable.

The nursing instructor explains that sometimes a nurse uses a mnemonic, such as OLDCARTS, as the nurse completes the assessment. What is the purpose of the mnemonic? A. To remember the elements that are important to assess with a symptom B. To remember the parts of a focused assessment C. To remember the order of the assessment D. To remember how to document assessment findings

ANS: A Feedback: Some providers use a mnemonic to remember the elements that are important to assess for the presenting symptom. OLDCARTS is one example and stands for onset, location, duration, character, associated/aggravating factors, relieving factors, timing, and severity. OLDCARTS does not help a nurse remember the parts of a focused assessment, order of assessment, or how to document findings.

A student nurse is teaching a group of peers about vital sign changes in older adults. The nursing instructor should intervene when the student nurse states: A. "As a person ages, their pulse rate will naturally increase." B. "Respirations per minute may increase with age. " C. "Older adults have less subcutaneous fat, making their body temperature lower." D. "Elevated blood pressure is not a normal sign of aging."

ANS: A Feedback: The pulse rate of older adults takes longer to rise to meet sudden increases in demand, takes longer to return to resting state, and tends to be lower than that of younger adults. Hypertension is not considered a normal age-related finding. Typically, older adults have less subcutaneous adipose tissue, which results in a decrease in body temperature. Respiration rate typically does not change with age.

The ICU nurse is caring for a trauma victim whose status is critical. On assessment, the nurse notes uremic frost along the client's hairline. What would this indicate to the nurse? A. Renal failure B. Cardiovascular failure C. Hepatic failure D. Respiratory failure

ANS: A Feedback: Uremic frost is a sign of marked renal failure. This appearance results from precipitation of renal urea and nitrogen waste products through sweat onto the skin. Uremic frost is not related to cardiovascular failure, hepatic failure, or respiratory failure.

For what is light palpation appropriate? (Select all that apply.) A. Inflamed areas of skin B. Internal organs C. Skin texture D. Deep pain E. Surface lesions

ANS: A, C, E Feedback: Light palpation is appropriate for the assessment of surface characteristics, such as texture, surface lesions or lumps, or inflamed areas of skin (e.g., over an intravenous site). Light palpation is not appropriate for assessing internal organs or deep pain.

Why do nursing students review medical records? (Select all that apply.) A. To enhance clinical learning B. To compare nursing care provided to clients C. To evaluate the plan of care for a specific client D. To verify that laboratory results are accurate E. To better understand complex clinical situations

ANS: A, E Feedback: Students in various health care disciplines review client records to enhance clinical learning and to better understand complex clinical situations. They can access and review records during care delivery. Nursing students do not use medical records to compare nursing care provided to a client population, evaluate the care plan, or verify that laboratory results are accurate.

A pediatric nurse, working in a community health clinic, is about to start an interview with a school aged child. What is the most appropriate way to address this child and the accompanying adults? A. Call the child by their legal name and refer to the parents as Mr. and Mrs. B. Call the child by their first name and ask the parents how they prefer to be addressed C. Call the child by their first name and refer to the parents as Mr. and Mrs. D. Call the child by their full name and refer to the parents as "mom" and "dad"

ANS: B Feedback: The nurse should refer to children by their first names and ask the adults what name they prefer for address. The nurse avoids calling parents "mom" or "dad" to maintain professional communication. Referring to the adults as Mr. and Mrs. is making an assumption that may not be correct.

An older adult comes to the clinic reporting pain in the right lower quadrant. When assessing the client's pain, what elements would the nurse include? (Select all that apply.) A. Aggressiveness B. Intensity C. Quality D. Location E. Quantity

ANS: B, C, D Feedback: Basic elements of pain assessment include intensity, quality, and location. Other basic elements are location, duration, quality/description, alleviating/aggravating factors, pain management goal, and functional goal. Aggressiveness and quantity are not identified aspects of pain.

When considering the head and neck, what screening should be included as a component of a pregnant woman's regular examinations? A. B12 B. folic acid C. thyroid D. amniocentesis

ANS: C Feedback: A focus area for pregnant women is the need for regular examinations that include thyroid screening. Such prenatal care helps to ensure that thyroid levels remain within normal limits, protecting both the mother and the fetus. It is not generally recommended that pregnant women schedule screenings for B12 or folic acid. Amniocentesis is not a regular screening procedure.

What amount of weight loss over 36 months is associated with increased mortality among older adults living in the community? A. 1% B. 3% C. 5% D. 10%

ANS: C Feedback: A loss of as little as 5% of weight over a 3-year period is associated with increased mortality among older adults living in the community. This information makes the remaining options incorrect.

A nurse is conducting a mini-Cog examination (MMSE) on an older adult client. What is this nurse assessing for? (Select all that apply.) A. Disorientation B. Violence C. Recall D. Language E. Registration

ANS: C, D, E Feedback: The MMSE is used with adults to assess for cognitive impairment and its severity in orientation, registration, attention, calculation, recall, and language. Neither of disorientation and violence are assessed by the MMSE.

A nurse is caring for a client who belongs to the Mormon faith. What would be the best breakfast for this client? A. Coffee, scrambled eggs, and sausage B. Tea, pancakes, and a grapefruit C. Coffee, oatmeal, sausage, and an English muffin D. Orange juice, French toast, and bacon

ANS: D Feedback: Food practices may be based on religious beliefs, such as fasting or abstaining from eating certain foods. Mormons avoid coffee, tea, alcohol, and tobacco.

When percussing a client, where would the nurse expect to find the loudest tones? A. Over the liver B. Over the bladder C. Over the spleen D. Over the lungs

ANS: D Feedback: The loudest percussion tones are over the lungs and hollow stomach; the quietest tones are over bone. Solid organs such as the spleen and liver would not have loud tones when percussed. The bladder, if positioned to be percussed, would be full of fluid, which would not have a loud tone.

One of the goals of nursing is to provide care that is safe to clients. What is the best way for nurses to realize this goal? A. By accurately charting client care B. By continually assessing client laboratory values C. By continual communication with all members of the health care team D. By giving client care conferences including all members of the health care team

ANS: C Feedback: Nurses perform all these functions for clients. Nevertheless, the best way to provide safe client care is to continually communicate with all members of the health care team.

What is the use of dietary assessment data gathered from a client by a nurse? A. Initiate social service consult B. Evaluate an adequate diet C. Identify client outcomes D. Identify areas that are not of concern

ANS: C Feedback: Nurses use assessment information to identify client outcomes. The other options are distractors for the question.

A nursing instructor is discussing the purposes of health assessment. What is one purpose of health assessment? A. To establish a database against which subsequent assessments can be measured B. To establish rapport with the client and family C. To gather information for specialists to whom the client might be referred D. To quantify the degree of pain a client may be experiencing

ANS: A Feedback: A health assessment is performed to gain further insight into the current condition and to establish a database that subsequent assessments can be measured against.

The clinic nurse is educating a caregiver about promoting the development of a 2-year-old child. Which statement should be included in the teaching? A. "Allow the child to make choices when possible." B. "Engage the child in goal-setting activities." C. "Pick the child up when crying." D. "Encourage the child to play in large groups of children."

ANS: A Feedback: According to Erikson, caregivers must help toddlers learn how to discriminate and choose appropriately. Goal setting and learning to work with others are not priorities during the toddler years. Comforting a crying child to help that child develop trust is associated with infancy.

The nurse is caring for a female client with hormone disorder producing excessive testosterone. Which of the following is an expected finding when assessing this client? A. Hirsutism B. Rapid heart rate C. Sensitivity to cold D. Muscle cramps

ANS: A Feedback: Excessive androgenic hormones in a female client can increase testosterone levels and cause masculinization changes, including hair in male distribution patterns. This hair growth is called hirsutism. Muscle cramps and cold sensitivity are associated with decreased thyroid hormone levels, and a rapid heart rate is associated with increased thyroid hormone levels.

The nurse is conducting a skin assessment on a client and notices the client has bilateral patches on tops of both feet with no color. The nurse should document this finding as: A. vitiligo. B. erythema. C. pallor. D. tinea corporis.

ANS: A Feedback: Vitiligo is characterized by areas of no pigmentation. Erythema is redness from inflammation. Pallor is a generalized paleness in skin color often caused by anemia. Tinea corporis is characterized by a ring-like rash on the body.

The medical record serves many purposes. What are they? (Select all that apply.) A. Framework for medical information B. Means for financial reimbursement C. Research D. Care planning E. Information for the family

ANS: A, B, C, D Feedback: The medical record serves multiple purposes. In addition to being a legal document, the medical record is used for communication among health team members, care planning, quality assurance, financial reimbursement, education, and research. The purpose of the medical record is not for information for the family.

A nurse has assessed that a client's condition is worsening. The nurse is telephoning the primary care provider and providing an SBAR report about the client's condition. What would be important for the nurse to document? (Select all that apply.) A. The time B. Information from the previous shift C. The information received D. Specialists to whom the client is referred E. The name of the provider

ANS: A, C, E Feedback: It is important to document the call, including the time, who was called, what information the nurse gave to the provider, and what information the nurse received. It is not important to document assessment data from the previous shift or specialists to whom the client has been referred.

A nurse has been called to testify in a lawsuit brought by a client against his employer. This institution uses charting by exception (CBE). What type of legal problems does CBE pose? A. The charting format is not ethical B. Details are often missing C. Subjective information is often missing D. It reflects poor assessment skills on the part of the nurse

ANS: B Feedback: CBE may pose legal problems, because details are often missing. CBE does not omit subjective assessment, CBE is an ethical form of charting, and the question does not indicate that the assessment skills of the nurse are lacking in any form.

When a nurse asks a client "Do you have any thoughts of wanting to harm or kill yourself?" for what is the nurse assessing? A. Suicide attempts B. Suicide means C. Suicide risk D. Suicide plan

ANS: C Feedback: Suicide risk is assessed by asking, "Do you have any thoughts of wanting to harm or kill yourself?" This question does not assess attempts at suicide, means of suicide, or plans of suicide.

A nurse performs a comprehensive assessment on a client. Which is included only in a comprehensive assessment? A. Circulatory assessment B. Assessment of the airway C. Complete health history D. Disability assessment

ANS: C Feedback: The comprehensive assessment includes a complete health history and physical assessment. It is done annually on an outpatient basis, following admission to a hospital or long-term care facility, or as defined in a facility's standards of care in the acute care setting. Circulatory assessment, assessment of the airway, and disability assessment are part of an emergency assessment.

Student nurses are in the laboratory learning auscultation techniques. How would they be instructed to hold the chestpiece on the client? A. Place the endpiece between the thumb and the index finger B. Place the index and middle fingers on top of the stethoscope C. Place the endpiece between the index and the middle fingers D. Place the thumb and index finger on top of the stethoscope

ANS: C Feedback: When holding the chest piece, the nurse places the endpiece between the index and the middle finger, not on top of the stethoscope, which would distort the sound.

During the assessment of an adult woman at the clinic, the nurse finds indications that the client is a victim of abuse. When documenting findings, the nurse knows that it is important to be what? (Select all that apply.) A. Sympathetic B. Consoling C. Descriptive D. Non-biased E. Detailed

ANS: C, D, E Feedback: When documenting objective data, it is important to be detailed, be descriptive, and note findings without bias. Documentation should not be sympathetic or consoling.

During the Romberg test, a client is unable to stand with his feet together and demonstrates a wide-based, staggering, unsteady gait. The nurse would identify this as which of the following? A. Spastic hemiparesis B. Parkinsonian gait C. Scissors gait D. Cerebellar ataxia

ANS: D Feedback: A wide-based, staggering, unsteady gait and positive Romberg test (client unable to stand with feet together) suggest cerebellar ataxia. Spastic hemiparesis is characterized by a flexed arm held close to the body while the client drags the toes of the leg or circles it stiffly outward and forward. A Parkinsonian gait is a shuffling gait. A scissors gait is a short stiff gait with the thighs overlapping each other with each step.

A nursing instructor is describing the peripheral nervous system to a group of students. The instructor would explain that there are how many pairs of spinal nerves? A. 8 B. 11 C. 24 D. 31

ANS: D Feedback: The peripheral nervous system consists of 12 pairs of cranial nerves and 31 pairs of spinal nerves. The other options are distracters to the question.

A clinical instructor is teaching a nursing student group about organizing data when documenting and communicating assessment findings. The clinical instructor knows that the method being taught promotes critical thinking and clustering of similar data. The instructor is teaching about which type of assessment? A. Body systems B. Comprehensive C. Head to toe D. Emergency

ANS: A Feedback: A body systems approach is a logical tool for organizing data when documenting and communicating findings. This method promotes critical thinking and allows nurses to analyze findings as they cluster similar data. The comprehensive assessment is more encompassing in nature, including more aspects that the body systems approach. The head-to-toe assessment does not look at promoting critical thinking and clustering, rather going through a process to organize data in a logical fashion. The emergency assessment involves a life-threatening or unstable situation

Nursing students are learning about different methods of charting in the clinical laboratory. What method would they learn that is a shared mental model for improving communication between and among clinicians? A. SBAR B. CBE C. SOAP D. PIE

ANS: A Feedback: SBAR, first developed by Kaiser Permanente in Denver and supported by the Institute for Healthcare Improvement (IHI), is a shared mental model for improving communication between and among clinicians. SOAP, PIE, and CBE are not shared mental models.

Nurses weave the individualization of the client interview through all aspects of the encounter. The nurse should avoid assuming that clients follow cultural beliefs. In place of making this assumption, what should a nurse do? A. Assess the degree to which the client perceives the cultural beliefs B. Assess how acculturated the client is C. Know the mores of the culture D. Know his or her own cultural beliefs

ANS: A Feedback: The nurse should avoid assuming clients follow cultural beliefs and assess the degree to which each individual perceives those beliefs. Knowing the mores of the culture and the nurse's own cultural beliefs are important, but do not answer the question at hand. The nurse would have difficulty assessing how acculturated the client is within the client's cultural beliefs.

What is the most commonly accepted theory of pain? A. Pain stimulus theory B. Gate control theory C. Pain transmission theory D. Gatekeeper theory

ANS: B Feedback: Currently, the theory of pain with the widest acceptance is the gate control theory. The other three options do not represent a theory of pain.

What does the Health Insurance Portability and Accountability Act (HIPAA, 1996) regulate? A. Who will provide client care B. Privacy of information C. How insurance information is obtained D. Where a chart can be stored

ANS: B Feedback: HIPAA regulates the security and privacy of information. It does not regulate who will provide client care, how insurance information is obtained, or where a chart is stored.

Nursing students are in the laboratory practicing palpation. What would they learn about the best depth for moderate palpation? A. 1/2 to 1 cm B. 1 to 2 cm C. 1 to 2 1/2 cm D. 2 to 3 cm

ANS: B Feedback: Pressure during moderate palpation is firm enough to depress approximately 1 to 2 cm. Other options above would be either not deep enough or too deep to correctly demonstrate moderate palpation.

What type of family violence is among the most common type that children experience? A. Financial abuse B. Sibling violence C. Munchausen's syndrome D. Traumatic stress

ANS: B Feedback: Sibling violence is among the most common type of violence that children experience. The other options may be experienced by children but do not describe the most common type of family violence that children experience.

A client presents at the urgent care clinic with severe pain and pressure around the eyes. The nurse practitioner suspects a sinus infection. What is considered the typical standard diagnostic technique in evaluating sinus disease? A. transillumination B. computed tomography (CT) scanning C. percussion of the sinus cavities D. magnetic resonance imaging

ANS: B Feedback: The typical diagnostic technique in evaluating sinus disease is a CT scan. Historically, nurse practitioners assess the sinus cavities with transillumination, but this method has limited clinical significance and provides inconsistent results. The other options are not used to assess this condition.

What are the types of nursing assessments? (Select all that apply.) A. Physical B. Focused C. Mental D. Emergency E. Comprehensive

ANS: B, D, E Feedback: Three types of nursing assessments are common: emergency, focused, and comprehensive. Physical and mental assessments are areas addressed in the various types of nursing assessments.

What is a mandated goal included in Anderson and McFarlane's Community as Partner Assessment Model? A. Transcultural assessment of community allies B. Social assessment of community leaders as individuals C. Systemic evaluation to identify the effects of interventions D. Community evaluation to identify interventions in communities

ANS: C Feedback: Anderson and McFarlane's model mandates that every community assessment and intervention include systematic evaluation to identify the effects of interventions. None of the other statements accurately describe a mandated goal.

A newborn has a hemangioma on the face. What would be important for the nurse to include in client teaching? A. Will need surgery to remove B. Will become smaller over the first year of life C. Is made of epithelial cells that form caverns and fill with blood D. Will usually resolve by age 9 years

ANS: D Feedback: Hemangiomas are vascular lesions, present at birth, that rapidly develop and evolve into larger lesions but spontaneously resolve by age 9 years. The other options do not provide accurate information.

A client has sustained an injury to the cerebellum. Which area would be the primary area for assessment? A. Vital signs B. Neurologic system C. Cardiac function D. Coordination

ANS: D Feedback: The cerebellum's primary functions include coordination and smoothing of voluntary movements, maintenance of equilibrium, and maintenance of muscle tone. Therefore, a priority assessment area would be coordination. The other options listed are distracters.

A nurse is admitting a client, having completed the health history, and is now doing a physical assessment. The physical assessment will provide what type of data? A. Patient centered B. Subjective C. Unconfirmed D. Objective

ANS: D Feedback: The physical assessment follows the history and focused interview and includes objective data, which are measurable. Subjective data are gathered during the health history and generally provided by the client. All data collected regardless of method should be client centered. Unconfirmed data is common and will be evaluated through diagnostic studies regardless of how it was obtained.

The nurse suspects substance abuse in an adult client who has been admitted to the emergency department following a motor vehicle collision. What is the reason the nurse would administer the CAGE tool to this client? A. The CAGE tool tracks patterns over time B. The CAGE tool addresses problems at work C. The CAGE tool tracks the progress of the client's disease D. The CAGE tool addresses the client's denial

ANS: D Feedback: When screening for substance abuse, the client will most likely deny a problem. The CAGE tool is valuable because it addresses this denial. The CAGE tool does not address problems at work; it does not track the progress of the client's disease nor does it track patterns over time.

The nurse is assessing a client who is new to the unit. During inspection of the client, what will the nurse do? A. Tell the client that modesty is not necessary B. Make sure that the client is covered C. Look for internal abnormalities D. Smell for odors

ANS: D Feedback: Nurses perform inspection by consciously observing clients for physical characteristics and behaviors and smelling for odors. The nurse would explain to the client that every measure will be taken to protect modesty, the client will have to be uncovered for inspection, and internal abnormalities cannot be seen during inspection.

The nurse is estimating potential adult height of a female child. The father's height is 69 inches and the mother's height is 68 inches. What is the child's potential adult height in inches?

ANS: 66 inchesFeedback: There are formulas for estimating potential adult height for children (Leifer & Fleck 2013). For girls, the formula is father's height (inches) + mother's height (inches) - 5 inches/2.

The nurse is performing a health assessment on a new client. While taking the detailed history, the nurse knows to include what? A. Functional status B. Data focusing on the client complaint C. A focused assessment of the client complaint D. Family history for the past three generations

ANS: A Feedback: A detailed history includes data on all systems, psychosocial and mental health, and functional status. Data must be included information other than the client complaint. Family histories generally go back only to grandparents, not great-grandparents.

A nursing student is caring for an older adult from a different cultural background. The nurse's preceptor asks the student what would be an important assessment to make to provide quality nursing care for this client. What would be the student's most appropriate response? A. transcultural assessment B. respiratory assessment C. mobility assessment D. family assessment

ANS: A Feedback: A specific example of a comprehensive nursing assessment that attends to both social and cultural dimensions is the transcultural assessment. A respiratory assessment would be important if the client had a respiratory problem. Mobility and family assessments could be useful, but these aspects can also be addressed through a transcultural assessment.

The nurse is caring for a young child. The child draws a picture of their home and gives it to the nurse. Which is the correct documentation? A. The child is in the preoperational stage according to Piaget. B. The child is establishing trust versus mistrust per Erikson. C. The child has reached the concrete operation stage according to Piaget. D. The child is demonstrating generativity as described by Erikson.

ANS: A Feedback: According to Piaget, from ages 2 to 4 years, the child develops the ability to mentally represent an absent object. For example, the child can scribble designs that represent people, houses, cars, etc. At this developmental stage the child has reached the preoperational (not concrete) stage. The concrete stage generally occurs between ages 7 to 11 and is characterized by logical reasoning and understanding logical operations and being able to reverse or classify items. According to Erikson, trust versus mistrust is established in infants. The infant either comes to view others as trustworthy or develops a fundamental distrust of the environment. Also according to Erikson, generativity and intimacy versus isolation are associated with adults. These stages relate to establishing a concern for the next generation (generativity) and developing intimate and loving relationships with other people (intimacy).

The nurse is caring for a client with a nursing diagnosis of impaired skin integrity related to a stage III pressure ulcer. What would be the most important outcome for this client? A. The client exhibits no signs or symptoms of infection B. The client changes position every 2 hours C. The client keeps the area clean and dry D. The client knows prevention measures for pressure ulcers

ANS: A Feedback: All options are appropriate outcomes for this client, but the most important outcome is that the client exhibits no signs or symptoms of infection since infection is a risk for additional injury.

A clinical instructor is discussing with students the care provided to a client. The instructor asks the student why it is important to make timely entries into the medical record. What would be the student's best answer? A. To have up-to-date information on which to base clinical decisions B. To be able to verify what care has been given C. To communicate with other health care providers D. To be able to update the plan of care

ANS: A Feedback: All the responses are correct; however, the best answer is that prompt documentation allows health team members to use up-to-date assessment information to make clinical decisions.

The nurse is participating in a group assignment on performing a social assessment on the Hmong society. What would be important to include in this assessment? A. Societal trends and relationships B. Spiritual architecture C. Individuals in the society D. Occupational relationships

ANS: A Feedback: At the societal level, social assessment is intended to generate information about societal trends and relationships among social variables and prevalent health concerns. The other options are incorrect because an assessment of a society does not include spiritual architecture, the individuals in the society, or their occupational relationships.

A college football player has been hospitalized with knee surgery. When discussing dietary choices, what nutrient would the nurse encourage increasing to promote muscle healing? A. Protein B. Carbohydrates C. Grains D. Fats

ANS: A Feedback: Athletes may require additional protein for muscle building and maintenance; this client also requires protein for muscle repair. Hospitalized athletes and post-surgical clients do not generally require extra servings of carbohydrates, grains, or fats.

Piaget differentiates young adulthood from middle adulthood by the use of what function? A. Expertise B. Fluid intelligence C. Wisdom D. Cognitive pragmatics

ANS: A Feedback: Because expertise requires years of experience, learning, and work, middle adults are far more likely to have it compared to young adults. Therefore, the other options are distractors.

What is an excitatory process caused by pain stimuli that involves the spinal nerves and can persist when there is no longer stimulation? A. Central sensitization B. Neuronal windup C. Peripheral sensitization D. Neuronal plasticity

ANS: A Feedback: Central sensitization is an excitatory process involving the spinal nerves and produced by continued pain stimuli that can persist even after peripheral stimulation is no longer present. Neuronal windup is produced when repeated assaults on the afferent neurons create enhanced response and increased activity in the central nervous system. Peripheral sensitization is when peripheral nociceptors are sensitized to pain stimuli. Neuronal plasticity means the ability of the nervous system to change or alter its function.

A nurse is interviewing a client who uses an expression with which the nurse is unfamiliar. What is the most appropriate phrase for the nurse to use to clarify the expression's meaning from the client? A. "Tell me what you mean by ________?" B. "I think that expression means ____________." C. "That expression is unclear to me." D. "Where did you hear that expression?"

ANS: A Feedback: Clarification is important when the client's word choice or ideas are unclear. For example, the nurse states, "Tell me what you mean by _____." Another way to clarify is to ask, "What happens when you _____?" Such questions prompt clients to identify other symptoms or give more information so that the nurse better understands. The nurse can also use clarification when the client's history of illness is confusing. Asking where the client heard the expression is not an appropriate response to help clarify the expression. The nurse thinking or believing the expression means something does not clarify the true meaning of the expression with the client. The nurse stating that the expression is unclear does not clarify the meaning with the client.

When a client with opioid tolerance has an altered physiologic response to pain stimuli, the client develops a form of pain sensitivity called what? A. Opioid hyperalgesia B. Pain hyperactivity C. Opioid hypoalgesia D. Pain hypoactivity

ANS: A Feedback: Clients with a history of opioid tolerance pose difficult challenges to nurses for pain assessment. They have an altered physiologic response to the pain stimulus, and the repeated use of opioids causes their bodies to become more sensitive to pain. This sensitivity is called opioid hyperalgesia and can occur as soon as 1 month after opioid use begins. The other options are not terms used to describe pain sensitivity.

The nurse is reviewing the plan of care for a client with peripheral neuropathy. Which intervention by the client should the nurse be concerned about? A. Limit use of a heating pad to 15 minutes at a time. B. Inspect feet daily using a mirror. C. Test bath water temperature with the hands. D. Wear clean white cotton socks.

ANS: A Feedback: Clients with peripheral neuropathy can be easily burned by a heating pad that is too hot. The client should test bath water with hands first to ensure the temperature isn't too hot. Clients should inspect feet using a mirror if needed and wear white socks. This will enable the client to identify blisters, sores, cuts, or splinters that they might not be able to feel. Cotton socks will also help reduce friction on the feet and toes that could lead to blistering.

Which food is most appropriate for the nurse to recommend for a client who suffers frequent nosebleeds due to hereditary hemorrhagic telangiectasia? A. Vegetable omelet B. Garlic chicken C. Chocolate pudding D. Salad with ginger dressing

ANS: A Feedback: Dietary recommendations for this bleeding disorder include decreasing foods high in salicylates, such as red wine, spices, chocolate, coffee, and some fruits. Provide education about supplements with antiplatelet activity, such as garlic, ginger, ginseng, gingko, and vitamin E. A vegetable omelet would be the most appropriate food choice since it doesn't contain salicylates or antiplatelet supplements.

A nursing instructor is discussing techniques used in the inspection of a client. What would the instructor note as a priority when inspecting a client? A. Adequate exposure of the body part B. Dim lighting in the examination room C. Use of therapeutic touch when assessing the client D. Use of therapeutic communication when assessing the client

ANS: A Feedback: During inspection, adequate exposure of each body part is necessary. Concurrently, nurses take measures to maintain the privacy of clients through appropriate draping, especially over the breasts in women and genitalia in both men and women. Adequate lighting is essential to observe color, texture, and mobility. Therapeutic touch and therapeutic communication are important but not the priority when inspecting the client.

The nurse is assessing a client who appears to have a swollen thyroid gland (goiter). Which of the following is a priority question the nurse should ask the client? A. "Do you have any difficulty swallowing?" B. "Do you have difficulty sleeping at night?" C. "What type of table salt do you use?" D. "Has your weight changed over the last few months?"

ANS: A Feedback: Dysphagia (difficulty swallowing) may accompany a growth or lesion on the thyroid gland. Swallowing difficulties are common in hospitalized patients and may prolong the length of stay because of an inability to obtain adequate nutrition for healing. Difficulty swallowing can compromise the airway, making it the priority in the assessment. The other choices relate to the thyroid but are not a priority as they don't affect the client's airway.

Teenagers doing community service following arrest for driving under the influence and are working at the rehabilitation hospital with clients who have paraplegia. These clients have been paralyzed by drunk drivers. How would the nurses who care for these clients best use the time spent with these teenagers? A. Educating them about not drinking and driving B. Teaching them how to turn these clients every 2 hours C. Fulfilling the court requirements D. Keeping the shelves restocked

ANS: A Feedback: Education for high-risk groups about not driving while under the influence or sleepy is critical. The nurses working with these clients would not spend time with the teenagers teaching them how to turn the clients, fulfilling court requirements, or keeping the shelves restocked.

When explaining how the nurse would test graphesthesia, which of the following would the nurse include? A. Client will close the eyes and identify what number the nurse writes in the palm of the client's hand with a blunt-ended object B. Client is to identify the numbers of points felt when the nurse touches the client with the ends of two applicators at the same time C. The nurse will simultaneously touch the client in the same area on both sides of the body and the client will identify where the touch occurred D. The nurse will briefly touch the client and the client will need to identify where the touch occurred

ANS: A Feedback: Graphesthesia is the ability to identify what is being drawn on the client's body when the client's eyes are closed. Two-point discrimination is tested by having the client identify the number of points felt when touched with the ends of two applicators at the same time. Extinction is tested by simultaneously touching the client in the same area on both sides of the body at the same points and having the client identify the area touched. Point localization is tested by briefly touching the client and then asking the client to identify the points touched.

An adult arrives at the psychiatric clinic in an unkempt state. An assessment notes clubbed fingers and nicotine stained fingertips. The client's history includes emphysema. What nursing diagnosis might the nurse use in this client's plan of care? A. Poor cognition related to poor oxygenation B. Poor cognition related to primary disease process C. Poor cognition related to drug withdrawal D. Poor cognition related to side effects of medication

ANS: A Feedback: Hands may be indicators of health problems, smoking status, drug withdrawal, low blood glucose level, or side effects of medications. Clubbing is seen in clients with emphysema or who use recreational drugs with talc in them; poor oxygenation affects cognition. The primary disease process for this client at this time would be psychiatric; the scenario does not mention drug withdrawal or medications with side effects that might cause poor cognition.

A nurse is having difficulty eliciting a patellar reflex. Which of the following would be most appropriate for the nurse to have the client do? A. Lock the fingers together and pull against each other. B. Clench the jaw tightly. C. Squeeze a thigh with the opposite hand. D. Stretch the arms overhead.

ANS: A Feedback: Having the client interlock his or her hands is a reinforcement technique that aids in enhancing the reflex response in the legs. Clenching the jaw or having the client squeeze one thigh with the opposite hand would be a reinforcement technique for testing arm reflexes. Stretching the arms overhead would be both inappropriate and ineffective in facilitating this assessment.

The nurse is caring for a client in the health care provider's office. In reviewing the client's chart, the nurse recognizes the need for providing the client with additional education related to COVID-19 when noting which of the following about the client? A. Works in the service industry B. Lives in a high-income household C. Has a history of low blood pressure D. Eats a diet low in carbohydrates

ANS: A Feedback: Health disparities that came to the forefront during the COVID-19 pandemic included underrepresented groups and people living in low-income households who were more likely to work in the service industries that remained open during the pandemic. Having high income, low blood pressure, and a diet low in carbohydrates are not factors associated with increased risk for COVID-19.

The nurse is performing assessments on several male clients. The first client is 52 years old and African American. The second client is 54 years old and Mexican American. The third client is 50 years old and Native American. The fourth client is 60 years old and Caucasian. Which client would the nurse expect to be tallest? A. Caucasian B. African American C. Mexican American D. Native American

ANS: A Feedback: Height varies little among racial groups compared to other anthropometric measures. Height results from genetics, nutrition, and stressors. Mean height varies by gender. In men, Caucasians as a group are usually tallest, followed by African Americans, and then Mexican Americans.

The nurse is assessing a client in the clinic who reports hearing "voices." What would be the most important assessment to make? A. The nature of the voices B. How the voices affect the client C. If the client also sees things D. Whether others hear the voices

ANS: A Feedback: If the client confirms auditory hallucinations, it is important to ask about their nature. Are they hostile or critical? Do they "command" or tell the client to do things such as harm self or others? The other options are appropriate, but not the most important since they are not directly associated with client safety.

The nurse notes that a client's conversation is often incoherent and wandering, with thoughts that seem illogical and inconsistent. What would these abnormal indications signify to the nurse? A. The client is thinking less efficiently B. The client is delusional C. The client is hallucinating D. The client has schizophrenia

ANS: A Feedback: Illogical, incoherent, irrelevant, wandering, inconsistent, or concrete thought processes are abnormal indications that the client is thinking less efficiently. Nothing in the scenario indicates that the client has delusions or hallucinations. Schizophrenia is a psychiatric diagnosis that a nurse would not identify a client as having.

An adult is recovering from sinus surgery. The client reports being in "a lot" of pain. The physician has ordered morphine every 3 hours. What intervention would be most appropriate? A. Give morphine every 3 hours B. Reassess the client before giving morphine as ordered C. Use nonpharmacologic pain relief prior to giving morphine D. Wait until the client asks for pain medication before administering morphine

ANS: A Feedback: In a client recovering from surgery, pain control is of utmost importance. Pain medication should be given as ordered for the first 24 hours unless the client refuses it. The other options are distracters for the question.

The nurse is caring for a hospitalized client from a culture that believes that a diet low in fruit and vegetables and high in fat is healthiest. While writing the plan of care for this client, it would be important to include which of the following nursing diagnoses? A. Emotional distress related to hospital diet B. Spiritual distress related to prescribed diet C. Nutritional deficiency related to beliefs in societal diet D. Nutritional excess related to societal diet

ANS: A Feedback: In many cultures, ideal body weight is higher than medical experts recommend. Such cultures may not consider "dieting" healthy. People may prefer to consume foods high in fat, salt, and cholesterol and low in fruit and vegetables because they believe that it is best for their health. The most appropriate nursing diagnosis in this case involves the emotional distress the client will likely experience following the hospital diet. Regardless of the client's potential beliefs, the hospital diet would not pose a risk for a nutritional deficiency. While the client's diet may be excessive in certain nutrients, the nurse or a dietitian would have to do more extensive nutritional assessment to arrive at specific conclusions.

A nurse is preparing to admit a new client to the unit and is reviewing the client record chronologically. In what phase of the interview process are the nurse and the client? A. Pre-interaction B. Beginning C. Working D. Closing

ANS: A Feedback: In the pre-interaction phase, the nurse reviews the record chronologically to detect patterns of illness, such as declining functional status, and to identify how things fit together. Beginning, working and closing are all phases in the interview process. The beginning phase occurs when introductions are exchanged, creating a safe and relaxed environment for the interview to occur. The working phase is the phase where data, both subjective and objective, are collected. The closing phase is when a summary of the interview session occurs, allowing the client to ask any questions or verbalize any concerns.

What is the element of pain transmission that causes nociceptors to perceive a nerve impulse? A. Transduction B. Transmission C. Perception D. Modulation

ANS: A Feedback: In transduction, noxious stimuli create enough of an energy potential to cause nociceptors (free nerve endings) to perceive a nerve impulse. Transmission is when the neuronal signal moves from the periphery to the spinal cord and up to the brain. Perception is when higher areas of the brain perceive the impulse being transmitted as pain. Modulation is the action of inhibitory and facilitating input from the brain modulating or influencing sensory transmission at the level of the spinal cord.

When assessing a client, the first skill used is inspection. What purpose does inspection serve? A. Gathering information B. Feeling abnormalities C. Observing modesty D. Identifying internal abnormalities

ANS: A Feedback: Inspection is the first technique of the overall general survey and for each body part, because it provides so much general information. Inspection is the one technique that is performed for every body part and body system. During inspection, the nurse does not use the hands to feel anything. Inspection is not to observe the client's modesty or to identify anything internal. Feeling some abnormalities and identifying some internal abnormalities would not be inspection but rather palpation. Inspection does not help with observing modesty.

The nurse is teaching the family member of an older adult about changes associated with the body and aging. Which statement by the family member indicates a need for additional education? A. "Limited exercise is recommended due to decreased vital organ capacity." B. "The ability learn new things will remain intact." C. "Taking multiple medications from various health care providers can cause delirium." D. "Older adults are more likely to get pneumonia."

ANS: A Feedback: It is important for older adults to maintain function by keeping a healthy weight, exercising, and treating conditions that arise before they cause serious health effects. Older adults are more likely to get pneumonia because the immune system weakens with age, and older adults may have additional chronic illnesses such as chronic obstructive pulmonary disorder, making them more susceptible to pneumonia. Older adults are at a higher risk for taking too many medications, also called polypharmacy. Polypharmacy symptoms can be psychological including delirium, apathy, delusions, and agitation. Although it may take older adults longer to learn new materials or tasks, the ability to learn new things remains present.

An older adult client with COPD has come to the clinic for a routine follow-up visit. The nurse escorts the client to an examination room and measures vital signs. The nurse would expect the client's vital signs to be what? A. Higher than normal B. Lower than normal C. Within normal limits D. The nurse would not routinely take this client's vital signs

ANS: A Feedback: Many variables can lead to increased vital signs, including pain, stress, anxiety, activity, and chronic disease processes. It is imperative that nurses measure vital signs correctly and accurately, understand the data, and communicate the findings appropriately. COPD is often a result of smoking and likely result in an increase in vital signs. The client's vital signs would be assessed at each clinic visit.

15. What phrase describes the pain phenomena felt in rheumatoid arthritis and osteoarthritis? A. Neuronal windup B. Neuronal plasticity C. Peripheral sensitization D. Central sensitization

ANS: A Feedback: Neuronal windup is produced when repeated assaults on the afferent neurons create enhanced response and increased activity in the central nervous system. Windup can cause tissues in the affected area to become extremely sensitive to pressure in areas not identified usually as painful. Examples of windup include rheumatoid arthritis and osteoarthritis. Neuronal plasticity is the ability of the nervous system to change or alter its function. Peripheral sensitization is the result of inflammatory process that creates hypersensitivity to touch or pressure. Central sensitization is the excitatory process involving spinal nerves produced by continued pain stimuli that can persist even after peripheral stimulation is no longer present.

The nurse is conducting discharge teaching to the caregiver of an older adult who was hospitalized following a fall at home. Which statement by the caregiver indicates a need for additional teaching by the nurse? A. "Loss of sensation in the toes is an age-related change." B. "Standing up slowly is important because dizziness can cause falls." C. "Certain medications can cause muscle weakness." D. "Changes in vision such as decreased accommodation happen with aging."

ANS: A Feedback: Numbness, tingling, or loss of sensation in the toes are not normal age-related findings and should be assessed immediately. Older adults are more prone to having their blood pressure drop briefly upon standing. This drop in blood pressure may lead to dizziness and places the client at increased risk for falling. Older adults are often prescribed several medications and should be monitored for side effects such as muscle weakness. Vision changes such as a decline of accommodative ability are common in older adults and can also contribute to falls.

What is the most common use of SBAR? A. Contacting a provider regarding a client issue B. Structuring communications during handoff C. Delegating care to nursing assistants D. Expressing concern about a client's condition to the charge nurse

ANS: A Feedback: Nurses most commonly use SBAR when contacting a provider regarding a client issue. SBAR also can serve as a method for structuring communication during handoffs, when delegating care to nursing assistants, or when expressing concern regarding a client's condition to the charge nurse or manager; however, none of these is the most common use.

The nurse is assessing a client in the emergency department who appears to be confused and unable to follow commands. When obtaining the client's social history, which of the following questions should the nurse ask first? A. "Have you consumed any alcohol today?" B. "Do you live in low-income housing?" C. "Are you able to afford your medications?" D. "Are you married?"

ANS: A Feedback: Nurses should use their judgement regarding the appropriate timing of a social assessment considering the health encounter, and only ask the questions that are essential. Asking about drug and alcohol use is a priority as the client is confused and unable to follow commands. Determining housing status, ability to purchase medications, and marital status are not immediately essential questions to ask.

The nurse is developing a plan of care for an older adult who lives independently. Which of the following should the nurse include? A. Allow additional time for the client to perform activities of daily living. B. Develop educational materials for the client's family instead of the client. C. Repeat the directions for each task multiple times. D. Provide all written educational materials in large text.

ANS: A Feedback: Older adults take longer to perform simple tasks such as activities of daily living. The client should be given additional time to complete regular tasks. Despite age, older adults still have the ability to learn and teaching should be directed toward the client. Teaching may need to be slowed down but repeating the same thing over and over may not be necessary and can frustrate the client. In addition, the client may not teaching related to each task. What needs to be taught should be assessed and not assumed. In addition, not all older adults require print to be in large text; that is an assumption the nurse should avoid making.

A nurse is conducting an admission assessment on an older adult client with an intractable oral infection. The client tells the nurse that he cannot smell things as well as he once could. What should the nurse do initially? A. Recognize that the sense of smell decreases with age B. Conduct olfaction tests C. Notify the physician immediately D. Verify the client's ability to taste all flavors

ANS: A Feedback: Olfactory sensory fibers decrease after the age of 60 years. In the oral cavity, a thinning of the soft tissue of the cheeks and tongue results in an increased risk of ulcerations, infections, and oral cancers. The nurse would not conduct tests on the client's sense of smell or taste, nor would he or she notify the physician immediately.

Student nurses are doing clinical hours on the medical-surgical unit. What additional assessment should the student nurses make when they take client vital signs? A. Oxygen saturation B. Blood pressure C. Heart rate D. Mobility

ANS: A Feedback: Oxygen saturation is assessed in hospitalized clients. Assessing blood pressure and heart rate is a regular vital sign assessment, not an additional assessment. Mobility is a distracter for this question.

A nurse who suffers from a respiratory infection is preparing to perform a shift assessment on a client when the urge to cough occurs. What is the nurse's best action? A. Cough into the inner aspect of the elbow. B. Cough into the air away from the client toward the hallway. C. Cover the mouth and nose with her hands while coughing. D. Perform hand hygiene before coughing into hands.

ANS: A Feedback: Patients and other people with symptoms of a respiratory infection are asked to cover their mouths and noses with a tissue when coughing or sneezing; covering with a tissue is better than covering with the hands. Additionally, clients should dispose of tissues directly into receptacles and perform hand hygiene after, not before, hands have been in contact with respiratory secretions. If a tissue is not available, according to the CDC a person should cough into the inner aspect of the elbow.

The charge nurse is prioritizing the review of intake and output records. Which client's records should be reviewed first? A. a client who has burns to 60% of their body B. a client who underwent hip arthroscopy two days ago C. a client who was admitted with malignant hypertension D. a client who was admitted with acute pain caused by gout

ANS: A Feedback: Patients with burns, drains, wounds, or severe diarrhea need to have outputs closely monitored. Chronic conditions such as hypertension, gout, and hip arthroplasty do not place a client at high risk for alterations in hydration and nutrition status. These conditions aren't associated with large fluid shifts, swallowing problems, or inability to eat and drink.

If A is greater than B, and B is greater than C, then it must be true that A is greater than C. This is called what? A. Transitivity B. Seriation C. Formality D. Hypothetical-deductive reasoning

ANS: A Feedback: Piaget's concept of transitivity refers to the child's ability to consider such problems as if A is greater than B, and B is greater than C, then it must be true that A is greater than C. Seriation refers to the ability to arrange objects by quantitative dimensions. Formality is the rigid observance of rules of convention or etiquette. Hypothetical-deductive reasoning is a very important method for testing theories or hypotheses.

The nurse in a long-term care facility is caring for an older adult client who had a 7% weight loss over a two-year period. Which is an expected intervention based on the information? A. Assess the client's dentures for proper fit. B. Place the client on fall risk precautions. C. Ask the client to eat in their room to avoid distractions at mealtime. D. Teach the client to choose food low in protein.

ANS: A Feedback: Poor-fitting dentures can detract from the enjoyment of meals, and unless the situation is remedied there can be a further decrease in weight. Losing weight doesn't automatically cause an older adult to be at risk for falls. If the client is having difficulty with maintaining proper fluid status, they will be at risk for falls. Older adults need to consume diets high in protein. Mealtimes should be as enjoyable as possible.

It is necessary for the nurse to accurately describe the sounds heard while percussing a client. Which of the following is a subjective description of percussion sound? A. quality B. intensity C. duration D. pitch

ANS: A Feedback: Quality refers to the subjective description of the percussion sound, such as a low-pitched thud of short duration versus a drum-like sound with high pitch and long duration. The intensity of the sound is its loudness, the duration is how long the sound lasts, and the pitch of the sound is the frequency or how soon the sound oscillates. Intensity, duration, and pitch are objective, not subjective, in nature.

A clinical instructor is discussing with a clinical group how to take a history of the client's present illness. A student asks how to best guide the interview. What would be the instructor's most appropriate answer? A. Follow the cues of the client during the interview B. Use a written checklist to make sure you cover all necessary areas C. Use a head-to-toe approach to make sure you do not miss anything D. Use a focused approach, asking only about symptoms of the present illness

ANS: A Feedback: Regardless of the order of data, the nurse guides the conversation following the cues of the client and uses a mental checklist to ensure that he or she has assessed all categories before the end of history taking. The nurse would not use a written checklist during the interview, and the nurse would not use a head-to-toe approach when eliciting information about the present illness. The nurse also would not focus only on the symptoms of the present illness.

A nurse is caring for an older adult who reports that he is becoming "forgetful." The nurse explains that some parts of memory decrease with aging, and some do not. What would the nurse identify happens with aging to the type of memory that corresponds with the retrieval of facts, vocabulary, and general knowledge? A. Decreases minimally B. Decreases, but may be from slower processing speed C. Decreases, but the client may use better strategies to minimize the effects D. Shows slight or no decrease

ANS: A Feedback: Semantic long-term memory involves the retrieval of facts, vocabulary, and general knowledge and decreases minimally with aging. Option C is active processing of information while it is held in short-term memory. Option D is memory for things the person is presently and actively thinking about; option B is the recollection of past events and personally relevant information.

What was the primary reason that the American Nurses Association and other accrediting agencies developed the National Standards for Culturally and Linguistically Appropriate Services in Health Care? A. The composition of the United States is multicultural. B. Ethnicity in nursing is part of the holistic process. C. Unless cultural differences are addressed, many clients do not respond well to nurses. D. Nurses need to understand every aspect of various cultures to provide good care to all clients.

ANS: A Feedback: The American Nurses Association, Joint Commission, American Psychological Association, and other accrediting agencies direct nurses to acknowledge and address the biopsychosocial and spiritual needs of clients. This is because the composition of the United States is multicultural. Ethnicity is not part of the holistic process. Clients may not respond well to nurses even when cultural differences are addressed. It is necessary to understand cultural differences when providing nursing care to specific cultures, but no nurse can know every aspect of all the different cultural groups encountered.

The nurse is using the Brief Pain Inventory to help assess pain in a client. Which of the following clients will benefit from this assessment? A. client admitted with bone cancer B. client who had a recent appendectomy C. client who fell at home and now has a hip fracture D. client who had local anesthesia for a lumpectomy

ANS: A Feedback: The Brief Pain Inventory was first developed to measure pain in clients with cancer and has been found effective for use with those who have chronic pain. Pain following a lumpectomy, pain associated with a hip fracture, and pain following an appendectomy are representative of acute pain.

A new graduate nurse has just started working. The new nurse asks a more experienced nurse to explain SOAP charting. What would the experienced nurse explain that the A in SOAP stands for? A. Analysis of data to identify a problem B. Assessment of subjective data C. Assessment of objective data D. Articulation of the plan of treatment

ANS: A Feedback: The SOAP format focuses on a single problem and includes subjective (S) assessment findings, objective (O) assessment findings, analysis (A) of the assessment data to identify a problem or indicate whether the problem is improving or worsening, and plan (P) for treating or improving the problem. Assessment and articulation would be incorrect.

An adult woman presents with overwhelming fatigue, reporting "It seems like I sleep all the time and yet I am still tired" and "I never seem to have the energy to clean my house or cook dinner." Diagnostic studies show a TSH of 6.2 mU/L and a T3 of 65 ng/dL. What outcome might the nurse identify for this client? A. The client verbalizes increased energy B. The client participates in activities with no change in vital signs C. The client verbalizes desire to participate in physical activities D. The client sleeps 12 hours daily

ANS: A Feedback: The client verbalizes increased energy, and well-being would be the most appropriate outcome. Nothing in the scenario indicates that the client is having a problem related to her vital signs. The client is not having a problem with desire to participate in activities; the problem is her energy level. Twelve hours of sleep daily for an adult is excessive.

Student nurses are taking vital signs at a community screening clinic. The nursing instructor should intervene when the student nurse performs which of the following actions? A. Applies the cuff to the right arm above the client's shunt location. B. Places the arm on the table below the level of the heart. C. Obtains a larger blood pressure cuff for an obese client. D. Asks the client to sit quietly while performing the blood pressure.

ANS: A Feedback: The extremity with a shunt should not be used for obtaining blood pressure. The client should sit quietly and calmly, the arm should be at arm level, and an appropriately sized cuff should always be used when obtaining blood pressure readings.

A student nurse is taking a nursing test and is asked a question about the columella. What is the best description of the term columella? A. The structure that divides the nares B. The lining of the nares C. The lining of lymphatic ducts D. The mound of tissue just posterior to the eustachian tube

ANS: A Feedback: The midline columella divides the oval nares (nostrils), which are openings that lead into the internal nose and are lined with skin and ciliated mucosa. That lining is known as the vestibule. Just posterior to the eustachian tube opening is a mound of tissue known as the torus tubarius.

A couple adopts an 8-month-old infant. The clinic nurse writes a care plan that includes the diagnosis of risk for delayed child development related to recent adoption. What would be an appropriate nursing intervention for this infant/family? A. Consider visits by a home health nurse to assess the environment for safety and comfort B. Teach parents to discipline the infant C. Teach parents to stimulate the infant as needed D. Consider a visit by a home health nurse to assess parenting skills

ANS: A Feedback: The most appropriate intervention for risk for delayed child development related to recent adoption is to consider a home health nurse visit to assess the environment for safety and comfort. Infants are too young for discipline, and this has no connection to the nursing diagnosis. All children need stimulation. Nothing in the scenario indicates that the couple's parenting skills are in question.

The nurse is caring for an older adult client with a nasogastric feeding tube ordered by the physician. The nurse notes that the client is not a mouth breather and having no difficulty breathing. While inserting the feeding tube, the nurse encounters difficulty getting the tube through the nares. What should the nurse suspect? A. Deviated septum B. Obstructed turbinate C. Swollen nasal passages D. Hypertrophied adenoids

ANS: A Feedback: The nurse assesses the client's nares for patency when inserting a tube into the nose for feeding. A deviated septum or obstructed nares may make insertion difficult. The nares may be obstructed, but the turbinate would not be. If the client is breathing through the nose, the nurse would not suspect swollen nasal passages. The nurse would not suspect hypertrophied adenoids in an older adult client.

Upon entering an adult client's room to begin a shift assessment, the nurse should call the rapid response team based on which assessment finding? A. Systolic pressure 180 mm Hg. B. Apical pulse 70 beats/minute. C. Respirations 12 breaths/minute. D. Oxygen saturation 95% on room air.

ANS: A Feedback: The nurse may call a rapid response team if the client displays the following: respirations less than 10 breaths/min; oxygen saturation less than 92%; pulse less than 55 beats/min or greater than 120 beats/min; systolic blood pressure less than 100 or greater than 170 mm Hg.

When auscultating the client's lungs, how should the nurse position the earpieces of the stethoscope? A. Pointed slightly forward toward the nose B. Pointed slightly downward toward the occiput C. At a 90° angle from the nose D. At a 45° angle to the occiput

ANS: A Feedback: The nurse should tilt the earpieces slightly forward so that the point on the earpiece is forward in the same direction as the nose. The other options are incorrect. Not positioning the earpieces correctly can lead to a less than optimal auscultation of the client's lungs.

A community health nurse is planning individualized care for a community. What does the nurse use as a framework for this plan? A. Nursing process B. Diagnostic reasoning C. Critical thinking D. Community care map

ANS: A Feedback: The nursing process serves as a framework for providing individualized care not only to individuals but also to families and communities. Diagnostic reasoning, critical thinking, and community care maps are not frameworks for providing individualized care to a community.

A woman comes to the emergency department with a broken humerus. The client lives in an area with a high crime rate and low socioeconomic demographic. During the examination, the client states, "I was fighting with my husband. He pushed me and I fell, landing on the coffee table. That is how I broke my arm." Why might this client be so forthcoming in her disclosure of violence? A. Decreased social stigma about violence B. Low ethnic rate of acceptance of violence C. Increased cultural statistics on violence D. Decreased environmental pressure for violence

ANS: A Feedback: The social stigma attached to human violence may vary among cultures. For example, a client from a neighborhood with a high rate of violent crime may feel less stigmatized because it is a common occurrence.

A new graduate nurse, attending a hospital orientation, is asked to explain the goals of a cultural assessment. What would be the graduate nurse's best answer? A. Developing and implementing a culturally congruent plan of care B. Comparing social care needs of the specific person C. Acquiring knowledge about the community's cultural beliefs and practices D. Comparing social and health beliefs of public health agencies

ANS: A Feedback: The specific aim of cultural assessment is to provide an all-inclusive picture of the client's culture-based health care needs by (1) gaining knowledge about the client's cultural beliefs and practices; (2) comparing culture care needs of the specific person with the general themes of those of a similar cultural background; (3) identifying similarities and differences among the cultural beliefs of the client, health care agency, and nurse; and (4) generating a holistic picture of the client's care needs, upon which a culturally congruent nursing care plan is developed and implemented. The remaining options are associated with components of an effective cultural assessment.

The nurse is at a school health fair and is teaching a group of teenagers about preventing periodontitis. Which statement made by a teen indicates understanding of the teaching? A. "Vaping can lead to periodontitis." B. "Periodontitis can be prevented by getting sufficient sleep." C. "Periodontitis is primarily caused by lack of exercise." D. "Frequent sinus infections are indicative of periodontitis."

ANS: A Feedback: The use of smokeless tobacco such as snuff and dip and electronic cigarettes (vaping) is becoming more popular and has been linked to oral cancers, periodontitis, and tooth loss. Sinus infections, exercise frequency, and sleep are not associated with periodontitis.

A woman newly immigrated to the United States is admitted to the obstetric unit. While doing a transcultural assessment, how would the nurse individualize questions for this client? A. Assessing if the client speaks and understands English B. Realizing that some women are not allowed an education in their home country C. Requesting a professional translator fluent in the woman's language D. Directing assessment questions only to the client's partner or family members

ANS: A Feedback: To best address individualization during a transcultural assessment with this client, the nurse must determine the degree to which the client effectively speaks and understands English. The nurse would need this information prior to asking for a translator, assuming the client was not allowed an education, or talking only with the partner or family members.

A nurse is admitting a new client. The client is lying in bed. Where should the nurse be positioned? A. Seated in a chair at eye level with the client B. Sitting on the side of the bed, looking down at the client C. Leaning on the nightstand at eye level with the client D. Standing beside the bed, looking down at the client

ANS: A Feedback: To facilitate optimal eye contact, the nurse needs to be at eye level with the client. Those who stand while clients are in bed will be taller than clients, assuming a position of power. Thus, the nurse should be seated in a chair at eye level with clients who are in bed during interviews. Leaning on the nightstand and sitting on the bed do not promote therapeutic communication or professionalism.

Nursing students are learning about human violence. The instructor is explaining key differences among various forms and how the differences affect assessment and intervention. What would the instructor explain is a key difference between violence against older adults and intimate partner violence? A. Perpetrator B. Age C. Support people D. Family involvement

ANS: A Feedback: Violence against older adults is intentional or unintentional acts such as physical, sexual, psychological, and financial abuse and neglect. Intimate partner violence is between spouses or nonmarital partners, threatened or actual physical or sexual violence or psychological/emotional abuse, coercive tactics, or both when there has been prior physical and/or sexual violence. Violence against an older adult can be perpetrated by someone other than the intimate partner. The other options are distractors for the question.

The nurse is assessing the pain of an older adult client who is recovering from a right hip open reduction procedure. What element would the nurse know it is important to review to best understand the client's pain? A. Sleep patterns B. Family history C. Genetic history D. Elimination pattern

ANS: A Feedback: When assessing pain in older adults, the nurse should be sure to also review the effects of pain on diet, sleep, and mood. Unrelieved pain may lead to insomnia or depression and seriously affect the client's quality of life. It would not be necessary to assess the family history, genetic history, or elimination pattern to gain insight into the client's pain level.

The nurse is caring for a client brought to the emergency department with confusion, unilateral facial drooping, and right-sided weakness. The nurse knows there is a need to perform an abbreviated acute assessment. Which is not part of the abbreviated acute assessment? A. Cranial nerve testing B. Pupillary reaction C. Gross assessment of sensation D. Vital signs

ANS: A Feedback: When situations indicating the likelihood of a stroke are identified, rapidly assessing key areas included in the neurological examination is the first step in determining the nature of the problem and possible urgent interventions. This abbreviated acute assessment includes items such as assessing pupilary reaction, obtaining vital signs, and completing a gross assessment of sensation. It does not include testing each of the cranial nerves.

The nurse is teaching an older adult diagnosed with diabetes about the skin. Which of the following should be emphasized? A. A neuropathic ulcer can develop without feeling it. B. Skin collagen decreases with age. C. Wound healing becomes prolonged with age. D. Hydration alters skin turgor.

ANS: A Feedback: While all options are true of older adults' skin, it is most important to emphasize to a client with diabetes that a neuropathic ulcer can develop without the client feeling it. People with diabetes are more likely to experience decreased sensation in the feet. The slowing of the healing process that comes with aging is also exacerbated by diabetes, as circulation may be slowed and the rate in which nutrients are delivered to wounds becomes decreased. Therefore, to reduce the risk of client injury, the nurse should emphasize that the client with diabetes checks their feet on a regular basis.

The nurse is reviewing the chart of an 87-year-old female French-speaking client admitted to the hospital with abdominal pain. What should the nurse do prior to beginning a cultural assessment? A. Determine if a translator is needed. B. Request an order for pain medication. C. Apply personal protective equipment. D. Ensure the client is placed on fall precautions.

ANS: A Feedback: Without adequate communication, it is difficult to develop rapport or trust. The nurse should first determine what kind of language assistance will be required to effectively communicate with the client and family. The other options can be determined, if necessary, after completing a physical assessment. Being able to properly communicate takes priority in conducting a cultural assessment, especially in this non-emergent situation.

The nurse instructor is helping student nurses to better understand the body's main source of energy, carbohydrates. The nurse instructor determines that further education is needed when a student lists which of the following as simple carbohydrate(s)? (Select all that apply.) A. glycogen B. fiber C. glucose D. dextrose E. fructose

ANS: A, B Feedback: Simple carbohydrates include glucose, dextrose, fructose, galactose, sucrose, maltose, and lactose. Examples of simple carbohydrates include candy, sugary drinks, syrups, and table sugar. Complex carbohydrates include starch, glycogen, and fiber. Examples of complex carbohydrates include whole grains, legumes, vegetables, and fruits.

An older adult client confined to a wheelchair is admitted to the hospital after a fall down a flight of stairs. The client is found to have multiple abrasions and bruises in various stages of healing. Abuse is suspected, and an Elder Assessment Inventory is performed. For what risk factors is the nurse assessing? (Select all that apply.) A. Strained mental health of caregiver B. Financial issues C. Cognitive decline D. Independence E. Optimal physical health of caregiver

ANS: A, B, C Feedback: Factors that put older adults at risk include dependency, cognitive decline, strained mental or physical health of caregivers, and financial issues. A commonly used tool to screen for maltreatment in older adults is the Elder Assessment Inventory. Independence and optimal physical health of caregiver are issues not addressed by the assessment tool.

As people age, several neurological changes occur. Neurons, brain size, and neurotransmitters decrease. What are some of the results of aging on the neurological system? (Select all that apply.) A. Slower thought processing B. Reduced response to stimuli C. Delayed reflexes D. Inability to process nutrients E. More frequent seizures

ANS: A, B, C Feedback: Neurons of the central nervous system, brain size, and neurotransmitters decrease with aging. Results include slower thought processing, reduced response to stimuli, and delayed reflexes. Inability to process nutrients is not a neurological change related to aging. Seizures in any age group represent a medical disorder requiring attention from a health care provider.

What physical characteristics should the nurse describe when discussing the characteristics of fetal alcohol syndrome? (Select all that apply.) A. Microcephaly B. Small eyes C. Flattened upper lip D. Prominent cheekbones E. Macrocephaly

ANS: A, B, C Feedback: Physical manifestations of fetal alcohol syndrome include microcephaly, flattened cheekbones, small eyes, and a flattened upper lip. Children with fetal alcohol syndrome have multiple developmental and learning disabilities. Prominent cheekbones and macrocephaly are not associated with fetal alcohol syndrome.

The nurse is conducting a 24-hour dietary recall with a client. Which of the following questions are appropriate to ask? (Select all that apply.) A. "When was the first time you ate or drank anything yesterday?" B. "What was the next thing you had to eat or drink?" C. "What foods did you eat that you might have put condiments on?" D. "Did you eat three large meals yesterday?" E. "Were you hungry in the late afternoon?"

ANS: A, B, C Feedback: When conducting a 24-hour recall, the nurse should ask open-ended questions to obtain complete and accurate results. Closed-ended questions don't allow for elaboration or additional details.

The triage nurse suspects malnutrition in an older adult with altered mental status who has been brought to the emergency department by family members. What visible signs might the nurse have noticed that would lead to the suspicion of malnutrition? (Select all that apply.) A. Atrophied tongue B. Temporal muscle wasting C. Generalized muscle weakness D. Dry eyes E. Productive cough

ANS: A, B, C, D Feedback: Clinical findings of malnutrition can occur throughout the body. Visible signs include muscle wasting, particularly in the temporal area; muscle weakness and decreased muscle size; tongue atrophy; and bleeding or changes in the integrity or hydration status of the skin, hair, teeth, gums, lips, tongue, eyes, and, in men, genitalia. A productive cough is not a visible sign of malnutrition.

Student nurses are practicing taking comprehensive health histories from one another. Which statement(s) by the pretend client should be included as data in the comprehensive health history? (Select all that apply.) A. "I started coughing a week ago." B. "My right upper abdomen hurts." C. "The pain has lasted for more than six days." D. "The pain is sharp." E. "I've followed my normal diet all week."

ANS: A, B, C, D Feedback: Location, duration, intensity, description, aggravating factors, alleviating factors, and functional impairment are components of a comprehensive health history. What was eaten for the last several days does not fall into the definition of a comprehensive health history.

According to Piaget, which child activities are indicative of the preoperational stage of development? (Select all that apply.) A. Recalls visiting the zoo several days after the visit. B. Participates in pretend play. C. Recognizes equal volume in different shaped containers. D. Scribbles pictures of pets, clouds, and cars. E. Engages in hypothetical-deductive reasoning.

ANS: A, B, D Feedback: Piaget divided the preoperational stage into two substages: (1) symbolic function and (2) intuitive thought. The child in the symbolic function substage, which lasts roughly from ages 2 to 4 years of age, can now mentally represent an absent object. For example, he or she can talk about a grandparent's house for many days after a visit. Scribbled designs represent people, houses, cars, clouds, and other objects (Santrock, 2012). Using language and pretend play (Fig. 8.2) are other characteristics of this substage. Children understand defined operations, such as recognizing equal volume in different shaped containers, in the concrete operations stage. Formal operations stage encompasses hypothetical-deductive thinking.

Wisdom, as defined by Piaget, has many characteristics. What are they? (Select all that apply.) A. Focuses on important and moderately easy matters B. Involves superior knowledge, judgment, and advice C. Applies only to nonspecific situations D. Is easily recognized by most people E. Is employed for personal well-being

ANS: A, B, D, E Feedback: Wisdom refers to an expert knowledge system that appears to focus on important and difficult matters often associated with the meaning of life and the human condition; requires a superior level of knowledge, judgment, and advice; is associated with knowledge that has extraordinary scope, depth, and balance; and is applicable to specific situations. Wisdom combines mind and virtue (character) and is employed for personal well-being as well as for the benefit of humankind. Although difficult to achieve, wisdom is easily recognized by most people, and it represents the capstone of human intelligence.

When inspecting the hair, what would the nurse note? (Select all that apply.) A. Color B. Condition of hair shaft C. Length of hair D. Hair breakage of more than 6 hairs E. Hair shafts that are shiny

ANS: A, B, E Feedback: During hair inspection, the nurse notes color, consistency, distribution, areas of hair loss, and condition of the hair shaft. Length of hair and hair breakage of more than 6 hairs are not things the nurse typically inspects.

When a client responds to a question with a "yes" or "no" answer, what appropriate responses by the nurse encourage the client to elaborate? (Select all that apply.) A. Yes B. I see C. Um hum D. Go on E. Okay

ANS: A, C, D Feedback: These responses encourage clients to say more and continue the conversation. They show clients that the nurse is interested. The nurse may nod the head or say "Um hum," "Yes," or "Go on" to cue clients to keep talking. Responses of "I see" and "Okay" do not encourage elaboration by the client and are therefore incorrect.

The paramedics are called to a gym to see an individual who has been exercising and developed pain in the upper right quadrant of the abdomen. The initial vital sign reading indicates a pulse of 175 beats per minute. This pulse would be considered what? (Select all that apply.) A. Elevated due to pain B. Elevated due to dehydration C. Elevated due to anxiety D. Elevated due to activity E. Elevated due to weight

ANS: A, C, D Feedback: Variables that can affect vital signs include pain, stress, anxiety, and activity. The individual's weight is not mentioned in the scenario, nor is that the individual is dehydrated.

A nursing instructor is discussing areas of health promotion related to the nose, sinuses, mouth, and throat. What topics would the instructor be most likely to cite? (Select all that apply.) A. Tobacco use B. Risks for chronic obstructive pulmonary disease (COPD) C. Obstructive sleep apnea D. Alcohol use E. Oral health

ANS: A, C, E Feedback: Health promotion areas related to the nose, sinuses, mouth, and throat include tobacco use, obstructive sleep apnea, oral heath, and cancer. Alcohol use would not be cited as a health-promotion area for the nose, sinuses, mouth, and throat, nor would risks for COPD.

A nursing instructor is discussing nutrition screening and assessment with a clinical group. What would this instructor identify for the students as parts of a complete nutrition screening assessment? (Select all that apply.) A. Physical examination B. Dietary log C. Calorie count .D Focused history of common symptoms E. Serial laboratory values

ANS: A, D, E Feedback: Parameters for a complete nutrition screening assessment include a risk assessment, focused history of common symptoms, comprehensive nutritional history, physical examination, calculated measurements, and serial laboratory values (especially during times of high metabolic demand, such as fever, pain, or infection or during limited nutritional intake). Generally speaking, a complete nutrition screening assessment does not include a dietary log or calorie count.

What are the steps for pain transmission in the gate control theory? (Select all that apply.) A. Unrelieved painful stimulus on a peripheral neuron causes the "gate" to open B. The gate opens through repolarization of the nerve fiber C. The brain stem recognizes the stimulus as pain D. The pain stimulus passes down the afferent pathway E. The pain stimulus crosses the dorsal horn of the spine to the limbic system

ANS: A, E Feedback: The steps for pain transmission in the gate control theory are as follows: (1) Continued painful stimulus on a peripheral neuron causes the "gate" to open through depolarization of the nerve fiber. This is accomplished by ion influx and outflow. (2) The pain stimulus then passes from the peripheral nervous system at a synaptic junction to the central nervous system up the afferent nerve pathways. (3) The pain stimulus passes up through and across the dorsal horn of the spine to the structures of the limbic system and the cerebral cortex. (4) In the cerebral cortex, the stimulus is identified as pain and a response is created. The response, once generated, passes down the efferent pathways where reaction to the pain is created. The distractors listed all have an incorrect component in the option which was noted above in the explanation.

What does the nurse knows about normal blood pressure? A. Stays level throughout the day B. Follows a diurnal rhythm C. Rises with the early morning fall of blood glucose D. Follows the same cycle as the sun

ANS: B Feedback: A daily, circadian (diurnal) cycle of blood pressure occurs, with it increasing late in the afternoon and decreasing in the early morning. Blood pressure does not stay level throughout the day. Blood pressure does not rise with the early morning fall of blood glucose. Blood pressure does not follow the same cycle as the sun.

The nurse in the dermatology clinic is assessing an adult who has presented at the clinic with a lesion on the left inner thigh. The client tells the nurse that the lesion was discovered one month ago and no changes in the color or size of the lesion have been noted. What would be the most appropriate teaching subject for this client? A. Skin self-examination B. Signs and symptoms of melanoma C. Recognizing different types of lesions D. Protection from sun damage

ANS: B Feedback: A simple method is to use the ABCDEs of melanoma detection: Asymmetry, Border irregularity, Color, Diameter of more than 6 mm, Evolution of lesion over time. The other given options are correct, but the most appropriate response is teaching about melanoma.

When documenting the findings of a neurologic assessment, which of the following would be most important? A. Verify the data before documenting. B. Describe the client's response. C. Label the client's behavior. D. Record objective data primarily.

ANS: B Feedback: Although data verification is important, when documenting the neurologic assessment findings, it is important to describe the client's response rather than label the behavior. Subjective and objective data must be documented.

An older adult is admitted to the unit with abdominal pain. The nurse doing the admission assessment knows what about pain in older adults? A. Older adults have fewer nerve fibers; therefore, they feel less pain B. Older adults may be reluctant to report pain C. Older adults are always in chronic pain D. Older adults are stoic and expect to be in pain

ANS: B Feedback: Although pain is prevalent in older clients, some of them see pain as just part of natural aging. They may be reluctant to report pain, because they want their providers to consider them "good clients," or they may fear that complaints of pain may lead to costly tests or expensive medications that they cannot afford. Older adults do not have fewer nerve fibers, they are not always in chronic pain, and they are not always stoic or expecting to be in pain.

The nurse is caring for a newly admitted adult client. When performing the general survey of this client, the nurse knows that accurate anthropometric measurements provide critical information about what? A. safety B. state of health C. growth pattern D. past surgeries

ANS: B Feedback: Anthropometric measurements are the various measurements of the human body, including height and weight. They provide critical information about the adult's state of health. Accurate measurements do not provide critical information about safety, past surgeries, or growth pattern in the adult client.

An older adult client presents at the clinic with a painful ulcer on the left big toe. The client states that the ulcer is very painful and never seems to heal. An assessment of the ulcer shows a lesion with well-defined wound edges. When dependent, the base of the lesion is ruddy in appearance and exhibits signs of infection. What would the nurse suspect? A. Infected spider bite B. Arterial ulcer C. Infected tick bite D. Venous ulcer

ANS: B Feedback: Arterial ulcers are usually located distally, such as at the ends of the toes or fingers. Wound edges are sharply defined; the base is pale when elevated and appears ruddy when dependent. These ulcers may be deep, frequently infected, and painful; they exhibit minimal granulation tissue. Venous ulcers occur when chronic pooling of blood happens in the extremities. The wound bed is beefy red with profuse exudate noted. Venous ulcers are generally painless. A spider bite usually presents as erythematous and papular with a visible small puncture opening at the center. A tick bite often presents as a erythematous target-like lesion that appears at the site of the bite.

A teenage client with cancer asks the nurse what hair is made of. What would be the nurse's best answer? A. "Hair consists mostly of carbohydrates." B. "Hair consists mostly of protein." C. "Hair consists mostly of inorganic matter." D. "Hair consists mostly of cellular waste."

ANS: B Feedback: Body tissues such as muscles, bones, teeth, skin, and hair primarily consist of protein. This information makes carbohydrates, inorganic matter and cellular waste incorrect answers to the client's question.

Friends bring a young adult client to the emergency department after a fall down a flight of stairs at a party. The nurse notes that the client has very little muscle mass or fat stores on the client's body. What syndrome should the nurse suspect the client has? A. Ascites B. Cachexia C. Dehydration D. Bulimia

ANS: B Feedback: Cachexia means a highly catabolic state with accelerated muscle loss and a chronic inflammatory response. It is a distinct syndrome separate from bulimia with the production of proinflammatory cytokines that contribute to the breakdown of fat and muscle protein, causing loss of both muscle mass and fat stores. Ascites, bulimia, and dehydration are not syndromes and are therefore incorrect responses.

An obese adult client is undergoing a preoperative examination prior to having bariatric surgery. The client tells the nurse that he has a red sore in the groin area that appears to be spreading. The nurse assesses the lesion and finds a macular erythematous lesion with satellite pustules. What would the nurse suspect? A. Roseola B. Candida C. Pityriasis rosea D. Herpes simplex

ANS: B Feedback: Candida is a fungal infection commonly found in skin folds or generally warm and moist areas. Affected sites are the axillae, inframammary areas, and groin. Satellite pustules are commonly found surrounding the erythematous macular lesion. Roseola is a viral illness with a rash that presents as discrete macules and papules, with an area of pallor surrounding each lesion. Pityriasis rosea is a viral infection initially presenting with a herald patch described as a large oval hyperpigmented lesion with a fine scale. Smaller lesions appear in subsequent days. Herpes simplex are cold sores and present as vesicles on an erythematous base.

A client complains of recurring headaches that are worse when first waking in the morning and with coughing or sneezing. What would be the nurse's most appropriate action? A. Ask the doctor for an order for an MRI B. Perform a focused assessment C. Prepare the client for a spinal tap D. Perform a generalized assessment

ANS: B Feedback: Characteristics such as pain that is worse in the morning on awakening and precipitated or made worse by straining or sneezing (potentially elevated intracranial pressure) versus pain that is worse as the day progresses (more likely tension) indicate a need for a more focused assessment. Other listed options are not the most appropriate action for the nurse to take.

When charting by exception is used in a health care agency, the most important aspect of this method is what? A. Organizing new forms for the nursing staff B. Identifying the standards and norms for the institution C. Training new nurses in writing charting by exception notes D. Pulling together a group of experts to teach agency staff

ANS: B Feedback: Clearly identifying the standards and norms and educating all users takes time and significant commitment from the agency using charting by exception. Organizing new forms for the nursing staff, training new nurses, and acquiring teachers for agency staff may be important, but they are not the most important aspect of this method.

What collection of factors are the most common causes for visual hallucinations? A. Multiple sclerosis, alcohol withdrawal, and medication side effects B. Medication side effects, alcohol withdrawal, and Parkinson's disease C. Parkinson's disease, medication side effects, and amphetamine withdrawal D. Narcotic withdrawal, multiple sclerosis, and head injury

ANS: B Feedback: Common causes of visual hallucinations include side effects from medications, alcohol withdrawal, and Parkinson's disease. Generally, multiple sclerosis, amphetamine withdrawal, narcotic withdrawal, and head injury do not cause visual hallucinations.

A nurse is preparing to assess the cranial nerves of a client. The nurse is about to test cranial nerve I. Which of the following would the nurse do? A. Use a Snellen chart to test visual acuity B. Ask the client to identify scents C. Test extraocular eye movements D. Perform the Weber's test

ANS: B Feedback: Cranial nerve I is the olfactory nerve, which would be tested by having the client occlude one nostril and identify a scent. Using the Snellen chart tests cranial nerve II, the optic nerve. Testing extraocular eye movements evaluates cranial nerve III (oculomotor), cranial nerve IV (trochlear), and cranial nerve VI (abducens). The Weber's test evaluates cranial nerve VIII (acoustic/vestibulocochlear).

When assessing an adult client experiencing diarrhea, the nurse notes a round "moon" face, a buffalo hump at the nape of the neck, and a velvety discoloration around the neck. What is the possible cause of these signs? A. Myxedema B. Cushing's syndrome C. Scleroderma D. Bell's palsy

ANS: B Feedback: Cushing's syndrome, excessive production of exogenous ACTH, can result in a round "moon" facies, fat deposits at the nape of the neck, "buffalo hump," and sometimes a velvety discoloration around the neck (acanthosis nigracans). The scenario does not describe signs and symptoms demonstrated by a client with myxedema, scleroderma, or Bell's palsy.

An infant is born with microcephaly. The infant also has a depressed nasal bridge, low-set ears, and a protruding tongue. What nursing diagnosis would be appropriate for this client and the parents? A. Inability to suck related to protruding tongue B. Knowledge deficit related to trisomy 21 C. Impaired hearing related to low-set ears D. Altered breathing related to depressed nasal bridge

ANS: B Feedback: Down's syndrome (trisomy 21) is a congenital condition that results from either an extra chromosome 21 or a translocation of chromosome 14 or 15 with 21 or 22. Manifestations in the head and neck region include microcephaly, a flattened occipital bone, slanted small eyes, a depressed nasal bridge, low-set ears, and a protruding tongue. The scenario described does not indicate that the infant has difficulty hearing, breathing, or feeding.

A student nurse is spending clinical hours on the medical-surgical unit with an experienced nurse. The student is assessing a client using indirect percussion. The student hears what sound while percussing the center of the client's abdomen? A. Resonance B. Tympany C. Dullness D. Damping

ANS: B Feedback: During indirect percussion, it is easiest to go from the center of the abdomen with tympany and move upward to the liver where dullness is percussed. Tympany would be described as loud in intensity, with a high pitch. Dullness would be described as soft to moderate in intensity with a high pitch. Resonance would be described as a moderate to loud intensity with a low pitch. Damping is not a term used to describe percussion sounds.

A hospice nurse is admitting a new client who states, "I'm not religious but consider myself spiritual." What change in attitude has been noted when illness stresses such a client's beliefs and values? A. Religious activities may lose importance. B. Religious activities may take a central position in life. C. Religious activities may blend with national identity. D. Religious activities may become formalized.

ANS: B Feedback: Even when daily prayers or other religious practices are not a routine part of a client's life, they often take a central position during life transitions, such as loss of a loved one, an accident, or serious illness. None of the other options are as likely to occur.

llness affect the client's quality of life. What type of assessment is this nurse performing? A. Comprehensive B. Functional C. Emergency D. Focused

ANS: B Feedback: Functional health patterns are especially important to nursing because they focus on the effects of health or illness on a client's quality of life. The emergency assessment is urgent in nature, addressing an emergent issue at hand. The comprehensive assessment is detailed in nature, not focusing specifically on the impact of the illness on the client's quality of life. The focused assessment addresses a particular issue or concern, not looking at the impact of the issue or concern on the client's quality of life.

A client reports, "bugs are crawling under my skin." The nurse knows that this tactile hallucination is usually associated with what? A. Medication side effect B. Methamphetamine use C. Brain tumor D. Parkinson's disease

ANS: B Feedback: Hallucinogen and methamphetamine use is associated with tactile hallucinations. The other options do not describe situations or disease processes where the client would report tactile hallucinations.

The nurse begins the community assessment process for a small town with an assessment of what relationship? A. The community's social support and marriage rates B. The community's level of education and rates of chronic illness C. The community's living conditions and divorce rate D. The community's median family income and high school graduation rate

ANS: B Feedback: Ideally, the process of community assessment begins with an assessment of various social, economic, environmental, and quality-of-life health indicators and their relationship with the community's health concerns. Examples of findings from a community social assessment include the relationship between social determinants of health (family income, level of education, social support, and living conditions) and chronic illness. None of the other options focus on health-related issues.

What structure is found midline in the tracheal area just beneath the mandible? A. Cricoid cartilage B. Hyoid bone C. Thyroid cartilage D. Adam's apple

ANS: B Feedback: Important landmarks for the head and neck region are in the tracheal area. The usually palpable U-shaped hyoid bone is located midline just beneath the mandible. The large thyroid cartilage consists of two flat, plate-like structures joined together at an angle and with a small, sometimes palpable notch at the superior edge. Usually more prominent in males, the thyroid cartilage is also called the "Adam's apple." The palpable cricoid cartilage is a ringed structure just inferior to the thyroid cartilage.

The Joint Commission developed a National Patient Safety Goal. What is a requirement of this goal? A. Health care agencies need to standardize their charting B. Health care agencies need to develop a standardized approach to hand off communications C. Health care agencies need to conform to Joint Commission communication templates D. Health care agencies need to computerize medical records

ANS: B Feedback: In 2006, the Joint Commission developed a National Patient Safety Goal that required agencies to develop a standardized approach to hand off communications, including the opportunity to ask and respond to questions. The Joint Commission does not have communication templates or mandate computerized medical records or standardized charting.

When Erikson published his theory in 1963, he predicted that childhood ended when? A. At the end of grammar school B. Puberty C. When the person gets his or her first job D. Teenage years

ANS: B Feedback: In Erikson's 1963 model, puberty signals the end of childhood. Therefore, the other options are incorrect. End of grammar school and teenage years are not specific enough. Employment does not correlate with the end of childhood.

Which of the following assessment findings would lead the nurse to suspect that a client has Bell's palsy? A. Inability to detect sharp and dull stimuli B. Inability to wrinkle the forehead C. Closure of the affected eye from swelling D. Muscle spasm of the lower face on the affected side

ANS: B Feedback: Inability to close eyes, wrinkle forehead, or raise the forehead, along with paralysis of the lower part of the face on the affected side, is seen with Bell's palsy. Inability to detect sharp and dull stimuli is associated with lesions of the trigeminal nerve (CN V). Closure of the affected eye from swelling would suggest trauma. Paralysis, not muscle spasm, occurs with Bell's palsy.

Madeline Leininger formulated a theory that highlights what information? A. Social behaviors in a community B. Nursing behaviors necessary to carry out an effective cultural assessment C. Nursing behaviors that compare personal philosophies of life and spiritual beliefs D. Behaviors and skills necessary to carry out a community social assessment

ANS: B Feedback: Leininger's theory identifies the relationships between cultural variables and health and highlights the nursing behaviors and skills necessary to carry out effective cultural assessment. None of the other options accurately describe Leininger's theory.

The Joint Commission mandates that nurses assess and reassess a client's pain level. A nurse's healthcare facility mandates pain reassessment at 30 minutes for any drug given intravenously. This mandate is based on what? A. The research supporting intravenous medications given for pain take half as long to work as oral medications B. The time it takes a pain medication to decrease pain intensity C. The time it takes a pain medication to block pain in a client D. The median half-life of an intravenous pain medication

ANS: B Feedback: Most healthcare facilities have a standard time frame for reassessment, such as 1 hour for oral medication and 30 minutes for pain medication given intravenously. Standards are based on the time it takes a pain medication to provide a noticeable decrease in pain intensity. The mandate from The Joint Commission does not look at the half-life of the pain medication, because the half-life would differ from drug to drug, and no drug is listed in the question. The pain medication does not block pain, but decreases the pain intensity. Research does not support that intravenous medication take half as long to work as oral medication, because this information depends on the individual drug and the chemical makeup of the drug.

After assessing a new client, the nurse documents findings in the medical record. What is the best example of documenting normal findings? A. Clothing appears too large B. Nails are strong C. Hair is thin and appears oily D. Oral mucosa is pink with white patches

ANS: B Feedback: Nails are strong; is the documentation that represents a normal finding. Clothing that is too large might indicate weight loss. Thin, oily hair is not generally a normal finding, nor are white patches on the oral mucosa.

A nurse is caring for a client who reports constant pain. The nurse knows that constant pain can lead to the modification of the function of the nervous system, which can, in turn, lead to what? A. Neuronal windup B. Peripheral sensitization C. Neuronal plasticity D. Chronic pain

ANS: B Feedback: Once pain becomes a constant stimulus, the nervous system can modify its function (neuronal plasticity). In turn, this can lead to another phenomenon called peripheral sensitization, by which peripheral nociceptors are sensitized to pain stimuli. Neuronal windup is produced when repeated assaults on the afferent neurons create enhanced response and increased activity in the central nervous system. Chronic pain is referred to as persistent pain.

A female teenager comes to the clinic reporting excessive hair growth. She tells the nurse that she is teased a lot because of hair growing on her shoulders and back; the client also reports that hair is growing on her upper inner thighs. What would the nurse suspect? A. Endocrine disorder B. Ovarian dysfunction C. Hepatic dysfunction D. Chronic nephrosis

ANS: B Feedback: Ovarian dysfunction may be characterized by the presence of hair in the beard area, abdomen, upper back, shoulders, sternum, and inner upper thighs on female clients. Body hair is not a sign of endocrine, hepatic, or kidney disorders.

An oncology nurse is caring for a client newly diagnosed with an occult nasopharyngeal malignancy. Where would the nurse expect this malignancy to be? A. Osteomeatal complex B. Rosenmuller's fossa C. Kiesselbach's plexus D. Wharton's ducts

ANS: B Feedback: Posterior to the torus region is the pharyngeal recess or Rosenmuller's fossa, which is a common site of occult nasopharyngeal malignancies. The middle turbinate and middle meatus area are known collectively as the osteomeatal complex, which is also the most anatomically significant area involved in chronic sinusitis. The anterior portion of the nasal septum has a rich vascular supply known as Kiesselbach's plexus. The submandibular gland is beneath the body of the mandible. Its Wharton's ducts run deep to the floor of the mouth and open on both sides of the frenulum.

In what way would a nurse indicate to a client that the client's concerns are not worth discussing? A. By being empathetic B. By providing false reassurance C. By being sympathetic D. By giving unwanted advice

ANS: B Feedback: Providing reassurance is not a therapeutic response, because by providing reassurance the nurse unconsciously indicates to clients that their concerns are not worth discussing. Empathy is a therapeutic response to a client and is a positive interaction. Being sympathetic and giving unwanted advice are nontherapeutic responses, but they do not tend to imply that the client's concerns are not worth discussing.

While assessing respirations and heart rate in a school aged client, the nurse finds that the client's heart rate increases during inspiration and decreases during expiration. What would be the most correct way to document this finding? A. The client has an abnormal heart rate B. The client has a sinus arrhythmia C. The client's heart rate speeds up and slows down in a cyclical pattern D. The client has a cardiopulmonary disorder

ANS: B Feedback: Rhythm may vary with respirations, speeding up during inspiration and slowing down with expiration. This finding, common in children and young adults, is called a sinus dysrhythmia or sinus arrhythmia. This condition is not an abnormal heart rate or a cardiopulmonary disorder. Describing the finding as the heart speeding up and slowing down is not correct terminology to describe the finding.

When evaluating a client's risk for cerebrovascular accident, which client would the nurse identify as being at highest risk? A. 42-year-old Caucasian woman who smokes B. 68-year-old African American male with hypertension C. 55-year-old Caucasian male who has two beers a week D. 35-year-old African American who has sleep apnea

ANS: B Feedback: Risk factors include older adulthood (risk doubling each decade after age 55), male sex, African American race, hypertension, smoking, chronic alcohol intake (more than three drinks per day), and sleep apnea among others. In the clients listed, the 68-year-old African American male with hypertension has the greatest risk due to his age, race, and hypertension. The other clients would be at risk, but the risk would be less.

The nursing instructor is discussing the function of sebaceous glands in the body. What would the teacher explain as the purpose of sebum to the students? A. Assists in keeping the skin intact B. Assists in friction protection C. Assists in protection from infection D. Assists in keeping skin dry

ANS: B Feedback: Sebum, an oil-like substance, assists the skin in moisture retention and friction protection. Sebum does not assist in keeping the skin intact, protecting from infection, or helping to keep the skin dry.

How does the nurse use critical thinking when accurately assessing vital signs? A. Evaluating assessment techniques B. Developing nursing diagnoses C. Monitoring evaluations D. Planning assessment techniques

ANS: B Feedback: Several nursing diagnoses can be addressed under vital sign assessment. The nurse would not evaluate assessment techniques, monitor evaluations, or plan assessment techniques when assessing vital signs.

A court trial is being conducted over an incident in the operating room. How would the medical record best be used in this instance? A. To provide a record of the nurse's activities B. To provide a record of the actual events C. To provide a record of how the client was harmed D. To provide a record of the physician's activities

ANS: B Feedback: The client record serves as a legal document recording the client's health status and any care the client receives. While all answers are correct, the best answer is providing a record of the actual events.

Erikson theorizes that the school-aged child faces the task of industry vs. inferiority. What is the danger in this stage of development? A. The child will separate himself or herself from others to avoid commitment to intimacy B. The child will not be able to learn to use adult tools C. The child will struggle to choose an occupational identity D. The child will overmanipulate self and work to repossess the environment in a repetitive fashion

ANS: B Feedback: The danger in this stage is that the child will not be able to learn to use the adult tools and will feel a sense of inferiority and inadequacy. It is difficult for the child to be admitted to an adult role in society without the tools to deal with the technology and economy of the culture. "The child will separate himself or herself from others to avoid commitment to intimacy" represents the danger of the young adult stage of development. "The child will overmanipulate self and work to repossess the environment in a repetitive fashion" represents the danger of the toddler developmental stage.

The nurse is caring for a client who, on the continuum between wellness and illness, is moving toward illness and premature death. How would the nurse know this to be true? A. The client stops doing wellness-promoting activities B. The client develops signs and symptoms C. The client begins exercising D. The client verbalizes anxiety over the cost of medications

ANS: B Feedback: The person who moves toward illness and premature death develops signs, symptoms, and disability, which, unfortunately, is when most treatment occurs in the current health care system. The client may stop doing wellness-promoting activities and not tell the nurse of this fact, which makes "The client stops doing wellness-promoting activities" incorrect. "The client begins exercising" is incorrect because a client who begins exercising is moving toward wellness, not illness. "The client verbalizes anxiety over the cost of medications" is incorrect because the verbalization of anxiety over financial matters is not an indication of illness.

The nurse is assessing a client with a history of drug addiction. What will be helpful in determining interventions that will be most beneficial for providing adequate pain relief to this client? A. Gathering information that the client wants to share about his pain B. Using in-depth questions to collect significant data about the client's pain C. Collecting objective data that the client chooses to share D. Collecting subjective data that the nurse notes during assessment

ANS: B Feedback: Use of in-depth questions to collect all the significant data from the pain assessment will be the biggest help in determining what types of interventions will be most beneficial for providing adequate pain relief to the client. Objective data are not shared by the client, and subjective data are not what the nurse notes during the assessment-these are what the client shares about the pain. While assessing a client's pain, the nurse needs to gather more information than is freely shared by the client.

An older adult presents at the clinic with reports of a painful neck. On palpation, the nurse notes a hard, nonmovable mass, approximately 20 mm, that is painful to touch. The area seems to have several nodes matted together. How would the nurse chart this last finding? A. Nodes feel matted together on palpation B. Nodes are delimited on palpation C. Nodes appear grown together on palpation D. Nodes are demarcated on palpation

ANS: B Feedback: Usually, no lymph nodes are palpable in the adult. If a node is palpable, it is important to describe the following characteristics: location-which lymphatic chain and where along that chain is the node; size-in mm or cm; consistency-how hard or soft is the node; mobility-it should be freely movable; delimitation-there should not be any matting together of lymph nodes. The other options do not demonstrate accurate documentation.

According to the guidelines from the Centers for Disease Control and Prevention (CDC), why are nurses supposed to wear gloves? (Select all that apply.) A. To help maintain a sterile environment B. To reduce transient contamination of the hands C. To reduce the risk of infecting healthcare personnel D. To prevent the transmission of bacteria from nurses to clients E. To reduce the number of bacteria in the health care environment

ANS: B, C, D Feedback: The CDC recommends that nurses wear gloves to (1) reduce the risk of personnel acquiring infections from clients, (2) prevent the transmission of flora from health care workers to clients, and (3) reduce transient contamination of the hands of healthcare personnel by flora that can be transmitted from one client to another. Gloves do not help to maintain a sterile environment, nor do they reduce the number of bacteria in the health care environment.

How does computerized documentation enhance communication? (Select all that apply.) A. It is more complete than handwritten charting B. It is legible and time dated C. It permits multiple simultaneous users D. It increases compliance E. It verifies telephone communication

ANS: B, C, D Feedback: The computerized client record helps ensure client safety and enhances communication, because computerized documentation is legible and time dated, increases compliance, permits multiple simultaneous users, and permits surveillance of client data to identify clients at risk. Computerized client records are not more complete nor do they verify telephone communications.

What is the purpose of the tongue? (Select all that apply.) A. Provides resonance to the voice B. Manipulates solids and liquids when eating and drinking C. Helps with the identification of sweet, sour, salty, and bitter tastes D. Assists in speech production E. Aids in the production of saliva

ANS: B, C, D Feedback: The tongue manipulates solids and liquids in chewing and swallowing. It is also involved in speech production and taste. The anterior two thirds of the tongue surface contain taste buds known as vallate papillae, which identify sweet, sour, salty, and bitter tastes. The other options are distracters for the question.

What qualities does the type of aggression called "punking" or "bullying" have? (Select all that apply.) A. It is usually perpetrated by a male upon a female. B. The behavior is intended to shame the victim. C. The behavior can be both physically and verbally abuse. D. The behavior occurs in public. E. Is common in middle school and high school.

ANS: B, C, D, E Feedback: Punking and bullying are common among middle and high school males, usually resulting in the victim's shame, humiliation, and anger. Similar to bullying and sometimes used interchangeably, punking is a practice of verbal and physical violence, humiliation, and shaming, usually done in public or with an audience. Punking and bullying do not always involve opposite-gender relationships and do not represent humorous behaviors.

A client presents at the emergency room reporting "the worst headache I have ever had." What are the critical initial nursing interventions for this client? (Select all that apply.) A. Scheduling a magnetic resonance imaging (MRI) scan B. Physical examination for neurologic changes C. Scheduling a computerized tomography (CT) scan D. Conducting a focused history interview E. Scheduling an electroencephalography (EEG)

ANS: B, D Feedback: A client with severe headaches may be unable to provide a complete history, but a focused history and physical examination looking for neurologic changes are critical nursing interventions. Nursing interventions do not include scheduling MRIs, CT scans, or EEGs without a prescription.

An older adult client is admitted to the hospital with pneumonia. While performing the admission assessment, the nurse finds a reddened area on the client's coccyx. What would the nurse include about this finding in notes? (Select all that apply.) A. Depth B. Location C. Other lesions on body D. Size E. Texture

ANS: B, D, E Feedback: A wound is assessed for location, size, color, texture, drainage, wound margins, surrounding skin, and healing status. When documenting a lesion, the nurse would not address other lesions on the body or the depth of the lesion.

What steps are involved in the client-to-client transmission of pathogens? (Select all that apply.) A. The nurse uses an alcohol-based hand sanitizer for hand hygiene B. Organisms are transferred from the client to the nurse's hands C. Organisms survive on the nurse's hands for less than 1 minute D. The nurse's contaminated hands come into direct contact with another client E. Organisms are present in the client's immediate environment

ANS: B, D, E Feedback: Patient-to-client transmission of pathogens requires five sequential steps: (1) Organisms are present on a client's skin or immediate environment; (2) Organisms are transferred from the client to the nurse's hands; (3) Organisms survive on the nurse's hands for at least several minutes; (4) The nurse omits or performs inadequate or inappropriate hand hygiene; (5) The nurse's contaminated hands come into direct contact with another client or environment in direct contact with the client. The use of alcohol-based hand sanitizer breaks the five-step process. Organisms can live on the nurse's hands for more than 1 minute if not cleansed appropriately.

What risk factors should the nurse include in a discussion on the occurrence of neck cancer? (Select all that apply.) A. Female gender B. Male gender C. Coffee drinker D. Tobacco use E. Age older than 50 years

ANS: B, D, E Feedback: Risk factors for neck cancers include male gender, age older than 50 years, tobacco use, and alcohol consumption. For clients with such risk factors, nurses should especially emphasize teaching related to smoking prevention or cessation. Risk factors do not include female gender or being a coffee drinker.

An ICU nurse is caring for a client in shock. To obtain an accurate set of vital signs, the nurse uses a Doppler to obtain pulse and blood pressure. What would be the best technique for the use of the Doppler on this client? (Select all that apply.) A. Hold the probe firmly against the skin at the expected pulse site B. Hold the probe perpendicular to the skin at the expected pulse site C. Moisten the skin with water prior to using the probe D. Mark the location of the loudest sound E. Move the probe slowly over the area of the expected pulse site

ANS: B, D, E Feedback: The procedure for assessment of the pulse using a Doppler is as follows: (1) Apply the gel that is specific for the Doppler to the transducer probe. (2) Turn the Doppler on. (3) Adjust the volume. (4) Touch the probe lightly to the skin at the expected pulse site. (5) Hold the probe perpendicular to the skin and move it slowly where you anticipate that the pulse should be until it is located. (6) Wipe off the gel and mark the location of the loudest sound with indelible ink. Water cannot be used to moisten the skin prior to using the probe. An accurate reading will not be collected if the probe is pressed firmly against the skin.

The nurse performs a neurological assessment and determines the Glasgow Coma Scale (GCS) score is 15. What is the nurse's best action? A. Notify the healthcare provider. B. Re-assess in 15 minutes. C. Document the findings. D. Ask the client to open eyes on command.

ANS: C Feedback: A GCS score of 15 is the maximum score indicating the client's neurological status is normal. Therefore, the nurse should document the findings. This information makes all the remaining options incorrect.

The nursing instructor is teaching about health assessment and explains to students how to assess the roles and relationships of the client. The students know that this type of information is assessed in what type of assessment? A. Body systems B. Head to toe C. Functional D. Focused

ANS: C Feedback: A functional assessment focuses on the patterns that all humans share: health perception and health management, activity and exercise, nutrition and metabolism, elimination, sleep and rest, cognition and perception, self-perception and self-concept, roles and relationships, coping and stress tolerance, sexuality and reproduction, and values and beliefs (Gordon, 1987). The body systems, the focused nor the head to toe assessment addresses the holistic needs of the client. The roles and relationships of the client would not be included in these assessment

When a nurse works in a health care agency that charts by exception (CBE), the nurse knows that the client assessment is structured by what? A. Medical diagnoses B. Client needs C. Standardized norms D. Joint Commission standards

ANS: C Feedback: A group may develop the standard of what it considers "normal" in each area of assessment (e.g., respiratory, mobility, psychosocial). These norms structure the client assessment. Client assessments are not structured by Joint Commission standards, client needs, or medical diagnoses.

In what life stage, defined by Erikson, is group identity important? A. Early adulthood B. School age C. Adolescence D. Young adult

ANS: C Feedback: Adolescents tend to cling together in cliques and crowds. Doing so helps to protect against the loss of identity through the assumption of a group identity that temporarily defines for the adolescent how to dress, act, and belong. Erikson explained that this behavior will eventually fall away as the individual defines his or her own identity. The other options are distractors for this question.

A nurse is preparing an educational event for the parents of children with respiratory disorders. What would the nurse tell the parents about allergies? A. If a parent has allergies, the child has a 25% chance of developing them as well B. Children will outgrow their allergies C. Allergy can affect any target organ in the body D. There are few effective treatments for allergies

ANS: C Feedback: Allergy can affect any target organ in the body. The nose and respiratory mucosa are the entry port for inhalant allergens. Thus, the nose and respiratory tract are common targets for inflammatory responses from allergen exposure. If one parent has allergies, the child has a 50% chance of developing allergies. Many allergies are lifelong problems; allergies can be managed and treated.

A client presents at the clinic reporting a possible sinus infection. How would the nurse assess the sinuses in this client? A. Indirect percussion B. Palpation C. Direct percussion D. Auscultation

ANS: C Feedback: An example of direct percussion is percussion of the sinuses in clients with sinus infections or percussion of the thorax in newborns to assess the air-filled lungs. The nurse would push or tap directly on the sinus area of the face with 1 to 2 fingers to assess pain or tenderness to the client. The nurse would not auscultate or palpate to assess the sinuses. The nurse would not use indirect percussion on the sinuses. With indirect percussion, the nurse assesses the density of the underlying tissue or organs to determine the perimeter or outline of the underlying structure. The indirect percussion method uses two hands to tap over the surface area to be assessed.

A nurse is performing an admission assessment on a new client to the unit. What would be the best way to phrase a question about the client's marital status? A. "Is your spouse living with you?" b. "Are you living with your spouse?" C. "Do you live alone or with someone?" D. "Are you married, divorced, or widowed?"

ANS: C Feedback: An inclusive, sensitive, and ultimately better question by which to determine the client's marital status is, "Do you live alone or with someone?" This phrasing provides a more direct avenue for finding out about support at home. The other options list questions that would not be the most appropriate question to ask the client about their marital status since they make assumptions or limit possible answers.

A woman and her teenager have come to the clinic. The teenager has acne lesions and says that the lesions are not well controlled. The mother asks the nurse what causes acne. What would be the nurse's best response? A. Acne is caused by the apocrine glands B. Acne is caused by decreased activity of the sebaceous glands C. Acne is caused by the impedance of sebum secretion onto the skin's surface D. Acne is caused by enlarged apocrine glands

ANS: C Feedback: As children approach puberty, the apocrine glands enlarge and become active. At puberty, sebaceous glands increase activity, resulting in large amounts of sebum secreted into the hair follicles of the face, neck, chest, and back. Anything impeding sebum secretion onto the skin's surface may result in the formation of closed comedones and ultimately acne.

The nurse enters a client's room to administer a prescribed anticoagulant for atrial fibrillation. The client exhibits new onset facial drooping and slurred speech. What is the nurse's priority action? A. Administer the PO anticoagulant immediately. B. Assess the client's bleeding time before medication administration. C. Ask the client to raise both arms in front of the client's body. D. Assess the client's vital signs and cranial nerves.

ANS: C Feedback: Atrial fibrillation increases risk for stroke because quivering atria can lead blood to stagnate and then form small clots. A clot that breaks off can circulate to the brain and block the artery, causing an embolic stroke. In sickle cell disease, blood cells tend to be stickier, causing clots to form more easily in narrowed arteries. This client's symptoms are consistent with a possible stroke. The nurse's priority is to perform a brief focused assessment and notify the healthcare provider right away. Administering the medication and checking the labs are not priorities of care during the acute phase of a stroke. Vital signs should be assessed right away, but it is not necessary to test cranial nerves initially.

The nurse is presenting a class for new mothers and explains that from ages 4 to 8 months infants perform secondary circular reactions. This occurs when A. The infant reproduces an event that initially happened by chance B. The infant coordinates schemes and intentionality C. The infant becomes more object-oriented, moving beyond being preoccupied with the body D. The infant experiments with new behavior

ANS: C Feedback: Between ages 4 and 8 months, infants engage in secondary circular reactions, by which they become more object-oriented and move beyond being preoccupied with the body. The infant coordinates schemes and intentionality occurs during coordination of secondary circular reactions. The infant reproduces an event that initially happened by chance represents primary circular reactions. The infant experiments with new behavior occurs during tertiary circular reactions.

The nurse assesses brisk reflexes in a client. The nurse would document this finding as which of the following? A. 1+ B. 2+ C. 3+ D. 4+

ANS: C Feedback: Brisk reflexes or reflexes that are more active than normal are documented as 3+. Reflexes that are decreased and less active than normal are documented as 1+; reflexes that are normal are documented as 2+; reflexes that are hyperactive and very brisk with rhythmic oscillations are documented as 4+.

What intervention would be most helpful when conducting an interview with a client who has stated, "I'm a little hard of hearing"? A. Asking the client if they are wearing a hearing aide B. Using pre-written cards that state the interview questions C. Closing the door may help to limit background noise. D. Introducing hand gestures whenever it is appropriate

ANS: C Feedback: Closing the door may help to limit background noise, making it easier for the client to hear. Not all clients with minimal hearing loss wear hearing aids. Pre-written questions and hand gestures are interventions reserved for those diagnosed with severe hearing limitations.

When caring for a client from a culturally different background, what is the goal for incorporating the client's health beliefs and practices into the nursing plan of care? A. To enhance the client's social system B. To enhance cultural connectedness C. Improvement of the client's health outcomes D. Improvement of communication with the client and family

ANS: C Feedback: Consideration of the client's cultural background and incorporating health beliefs and practices in care plans contribute to enhanced client experiences with health care and improve health outcomes. Incorporating the client's health beliefs and practices will not enhance this client's social system or cultural connectiveness, nor will it improve communication with the family.

What is the current focus of pain research? A. Elements that can affect the intensity of pain B. Stopping the cause of pain C. Elements that can affect pain inhibition D. Stopping the sensation of pain

ANS: C Feedback: Current research focuses on those elements that can affect pain inhibition and stop the pain stimulus. The other options listed are not where current research on pain is focusing.

A nurse in the emergency department is assessing a client admitted with suspected appendicitis. The nurse would avoid using which kind of palpation over the right lower quadrant of this client? A. None B. Light palpation C. Deep palpation D. Moderate palpation

ANS: C Feedback: Deep palpation should not be used over areas that pose a risk of injuring clients, such as over a suspected enlarged spleen or inflamed appendix.

The nurse notes an acute change in an adult client's mental status. The client has gone from being coherent and oriented to person, place, and time to having very disorganized thought processes. The client has become delirious. What are the possible effects of delirium on the client? A. Visual hallucinations B. Depression C. Increased risk for suicide D. Stomach pains

ANS: C Feedback: Delirium usually has an acute onset, and disorganized thoughts can place the client at risk for injury or suicide. Delirium generally does not cause visual hallucinations, depression, or stomach pains.

When taking a dietary assessment on a newly admitted client who has had a mild myocardial infarction and is a vegetarian, what would the nurse identify as being important to include in dietary teaching? A. Food/drug interaction with chlorophyll B. Food/drug interaction with melatonin C. Food/drug interaction with green leafy vegetables D. Food/drug interaction with grapefruit

ANS: C Feedback: Diet and food intake affect medications. Dark green leafy foods can decrease the effect of some anticoagulants, and many clients who have had a myocardial infarction usually take some kind of anticoagulant. Grapefruit, melatonin, and chlorophyll do not generally react with anticoagulants.

The nurse is caring for an adult client who presents at the clinic with reports of general malaise and fatigue. Physical assessment reveals that the client's lips are dry and cracked. What might this indicate? A. Heat stroke B. Viral infection C. Inadequate hydration D. Allergy

ANS: C Feedback: Dryness or cracking may indicate inadequate hydration. Lesions or aphthous ulcers may represent a viral infection. Swelling or edema of the lips suggests allergy. Heat stroke does not present with dry, cracked lips.

Students are learning about the many uses of the medical record. One of these uses is to perform an internal audit. What is the goal of an internal audit? A. The evaluation of financial reimbursement B. The evaluation of client nutrition C. The evaluation of care for continual improvement D. The evaluation of timely documentation of pain

ANS: C Feedback: During an internal audit, the goal is to evaluate the care provided for continual improvement. Other types of audits are conducted for information on financial reimbursement, client nutrition, and timely documentation of care provided.

A nurse is teaching a class on diet and nutrition to a group of mothers who are breast-feeding their infants. What would the nurse tell the group is the emphasis of nutritional guidelines? A. Decreased intake of grains B. Weight loss C. Variety D. Increased intake of meats

ANS: C Feedback: Emphasis of nutritional guidelines is on variety; increased intake of vegetables, fruits, lentils, and grains, particularly from plant sources; and meeting individual nutritional needs while avoiding either deficiencies or excesses in nutrient intake.

A nursing instructor is talking about nonverbal communication with the nursing class. The instructor explains that facial expressions should be what? A. Happy B. Inquisitive C. Relaxed D. Detached

ANS: C Feedback: Facial expressions should be relaxed, caring, and interested. Facial expressions that are happy, inquisitive, or detached can interfere with the therapeutic communication process.

When doing a risk assessment of the nose, sinuses, mouth, and throat, what finding might indicate an allergy? A. Low-grade fever B. History of macular rash C. Frequent childhood infections D. Family history of diabetes

ANS: C Feedback: Frequent upper respiratory infections in childhood should raise the nurse's suspicion of allergy. Chronic inflammation of the respiratory tract may lead to mucosal damage with resultant chronic infections (e.g., sinusitis). The findings of low-grade fever, history of macular rash, or family history of diabetes would not necessarily raise the suspicion of allergy.

What is the most common indication of nutritional status in infants and children? A. Appetite B. Number of wet diapers/day C. Growth D. Sleep pattern

ANS: C Feedback: Growth charts are commonly used to indicate nutritional status. As an indication of nutritional status, appetite, number of wet diapers/day, and sleep pattern are generally not used.

HIPAA gives clients greater control over their medical records. What else does HIPAA provide? A. Copying of medical records B. Education of lay people about medical records C. Client recourse if privacy protections are violated D. Legal use of medical records

ANS: C Feedback: HIPAA provides for client education on privacy protection, client access to medical records, client consent prior to disclosing information from the record, and client recourse if privacy protections are violated. HIPAA does not address copying of medical records, education of lay people about medical records, or legal use of medical records.

How do nurses facilitate the achievement of high-level wellness with a client? A. Encouraging the client to keep appointments B. Providing the client information on alternative treatments C. Promoting health in the client D. Providing good client care

ANS: C Feedback: High-level wellness is a process by which people maintain balance and direction in the most favorable environment. The role of nurses is to facilitate this achievement through health promotion and teaching. Nurses do not facilitate the achievement of high-level wellness by encouraging clients to keep appointments, providing information on alternative treatments, or providing "good" client care.

The nurse is admitting a client to the hospital following a motor vehicle collision in which alcohol may have been a contributing factor. What tool might the nurse use to assess whether alcohol is a problem in this client's life? A. MMPI B. ABCT C. CAGE D. HOPE

ANS: C Feedback: If alcohol use might be a problem, the CAGE is a quick first-step questionnaire to use as an assessment tool. The MMPI is the Minnesota Multiphasic Personality Inventory used to aide in diagnosing psychological problems. The ABCT is used for assessment of mental status. It includes Appearance (posture, movement, hygiene, and dress), Behavior (level of consciousness, eye contact, facial expressions, speech), Cognitive function (orientation, attention span, memory, judgment), and Thought processes and is not a tool. The HOPE tool is used for assessing spirituality.

Which information in the client's history is a priority safety concern? A. Cleft lip and palate repair surgery B. Diminished sense of smell and taste C. Recent immigrant complains of cough D. Removal of oral cancerous lesion

ANS: C Feedback: Incidence of tuberculosis is approximately nine times higher among U.S. immigrants (W.H.O., 2013). Diminished senses of smell and taste may produce safety risks but are not as high priority as the immediate threat of tuberculosis, which is contagious. Lack of smell and taste are not contagious conditions. Removal of a cancerous lesion in the past does not pose an immediate concern like tuberculosis. Cleft lip and palate surgery are not immediate safety concerns like tuberculosis.

A client with alcoholism is on a detoxification unit. What nutritional deficit would the nurse expect in this client? Vitamin C Vitamin A Vitamin K Vitamin D

ANS: C Feedback: It is common for clients with a high alcohol intake to be deficient in the B vitamins and vitamin K. These clients are not generally expected to be deficient in vitamins C, A, and D.

A client brought to the emergency department after a diving accident tells the nurse, "I think I hit a rock, and now I can't feel my arms or legs." A rapid physical assessment shows no other apparent injuries. What initial nursing intervention is essential to the safety of this client? A. Send for a CT scan immediately. B. Draw a basic metabolic panel to check for infection. C. Maintain immobilization of the spine until it is cleared of injury. D. Log roll the client to remove backboard, reducing pressure on the spine.

ANS: C Feedback: It is essential to keep the spine immobilized to prevent spinal cord injury, so immobilization devices should not be removed until the spine is cleared of injury. A CT scan is not appropriate for this client; he or she would have an MRI ordered. Blood work on this client would not be essential to this client's care. Log rolling the client to remove the backboard would be contraindicated until the spine is cleared.

The nurse is caring for an older adult client and is reviewing information obtained in the health history assessment. The nurse knows that it is important to identify the pattern of illnesses and recognize how they might be related because of what factor? A. Client is hospitalized B. Client presents as being stoic C. Client is of advanced age D. Client is chronically ill

ANS: C Feedback: It is important to identify the pattern of the illnesses and recognize how they might be related as the client is an older adult. The question does not state that the client is in the hospital, stoic, or chronically ill.

During a neck assessment, where would the nurse focus palpation of the thyroid isthmus? A. Just above the thyroid cartilage B. Between the thyroid and the cricoid cartilages C. Just below the cricoid cartilage D. In front of the sternocleidomastoid muscle

ANS: C Feedback: Just below the cricoid cartilage, the isthmus of the thyroid should be palpable as a smooth rubbery band that rises and falls with swallowing. The other options do not accurately describe the location of the isthmus.

The charge nurse on a geriatric unit should further educate a new staff nurse who makes which statement? A. Transmission of pain along A-delta and C fibers may be altered in older adults. B. There is little information about the effects of increased age on pain. C. Pain sensation is diminished in older adults. D. Older adults tend to be undertreated for pain.

ANS: C Feedback: Little is known about the effect of increased age on pain perception. No evidence suggests that pain sensation is diminished in older adults, which is a common misperception. Transmission along the A-delta and C fibers may become altered with aging, but it is not clear how this change affects the pain experience. Studies of sensitivity and pain tolerance have indicated that changes in pain perception are probably not clinically significant. Because older people are likely to experience more adverse effects from analgesia, especially from opiates, health care providers may undertreat pain in older adults.

A client arrives at the emergency department by ambulance after an accident while playing softball. The client's left leg is swollen and deformed. The client describes the pain as a 9 on a 10-point scale. When the nurse assesses the client's blood pressure, what would the nurse expect to find? A. The blood pressure is lower than normal B. There would be no need to assess the blood pressure C. The blood pressure is elevated D. The blood pressure is within normal limits

ANS: C Feedback: Many variables affect vital signs, including pain, stress, anxiety, and activity. Pain and anxiety can contribute to increased blood pressure. The nurse would not expect to find the blood pressure lower than normal or within normal limits with the client's report of pain as a 9 on a 10-point scale. It would be expected that the nurse would assess the blood pressure upon arrival to the emergency department for this client.

A nurse is caring for three clients whose care involves complex situations and multiple responsibilities. What is the key to resolving problems for this nurse? A. Diagnostic reasoning B. Physical assessment C. Critical thinking D. Nursing care plan

ANS: C Feedback: Nurses are frequently involved in complex situations with multiple responsibilities. They are required to think through the analysis, develop alternatives, and implement the best interventions. Critical thinking is the key to resolving problems. Diagnostic reasoning is important in developing diagnostic statements, not in caring for multiple clients with complex care needs. Physical assessment is important in the building the foundation of the nursing care plan. The nursing care plan directs the care that will be provided for the individual client, but does not address the needs of caring for multiple clients.

A new graduate nurse is inspecting a client. What is a challenge the nurse will face? A. Maintaining client modesty B. Learning how to perform inspection C. Identifying subtle differences D. Documenting what is normal

ANS: C Feedback: One challenge that beginners face is identifying details and subtle differences. Maintaining client modesty, learning how to inspect, and documenting normal findings are skills that nurses learn before they graduate from nursing school.

A caregiver brings a handicapped client to the ED reporting the presence of an altered level of consciousness and the client's refusal to eat. When the client is found to be severely dehydrated, the nurse suspects neglect and asks the caregiver several questions regarding the client's activities, diet, and care. The caregiver states, "I didn't know it could hurt him if he didn't drink anything." This is an example of what kind of abuse? A. Psychological B. Intentional C. Unintentional D. Direct

ANS: C Feedback: Physical, sexual, psychological, and financial abuse and neglect may be intentional or unintentional. The scenario does not describe psychological abuse or intentional abuse. Direct is not a term commonly used to describe abuse.

Who are the major perpetrators of human trafficking? A. Individuals who own and manage sweatshops B. Individuals who kidnap children to sell for adoption C. Individuals who own and manage commercial "sex trade" businesses D. Individuals who sell people for cheap labor

ANS: C Feedback: Reasons people are trafficked include sexual exploitation, forced marriage, and cheap labor for domestic or commercial purposes. Those who own and manage commercial "sex trade" businesses (i.e., forced prostitution, stripping, pornography, live-sex shows) are the major perpetrators of human trafficking. None of the other options pose the risk presented by the "sex trade."

The nursing instructor is discussing the different types of pain with the nursing class. What type of pain would the instructor explain originates from a specific site, yet the client feels the pain at another site? A. Chronic pain B. Cutaneous pain C. Referred pain D. Somatic pain

ANS: C Feedback: Referred pain originates from a specific site, but the person experiencing it feels the pain at another site along the innervating spinal nerve. Chronic pain is pain referred to as persistent. Cutaneous pain derives from the dermis, epidermis, and subcutaneous tissues and is felt at its origination. Somatic pain originates from skin, muscles, bones, and joints and is felt at its origination.

The nurse is performing a generalized assessment of an older adult. The nurse notes that the client's skin is thin and rough with abrasions. The client tells the nurse that it seems to take "forever" for scratches to heal, "a lot longer than when I was younger.". How would the nurse note these findings in the client's medical record? A. The client has abnormal thinning of skin B. The client's integumentary system is within normal limits C. The client states that wounds are taking longer to heal D. The client has an abnormal inability to maintain temperature

ANS: C Feedback: Replacement of the epidermal layer decreases with aging, resulting in rougher skin texture and prolonged time for wound healing. It would be incorrect to state that the client has abnormal thinning of skin from the information given in the question. The nurse would not document that the integumentary system is within normal limits if the client states the wounds take an excessive time to heal. There is no information in the question that addresses the client have an abnormal inability to maintain temperature.

A student nurse studying hypertension would learn that the risk factors include what? A. Family history, obesity, vegetarian diet B. Cigarette smoking, heavy alcohol consumption, hepatic disease C. Prolonged stress, renal disease, heavy alcohol consumption D. High cholesterol and triglyceride levels, family history, hepatic disease

ANS: C Feedback: Risk factors for hypertension include obesity, cigarette smoking, heavy alcohol consumption, prolonged stress, high cholesterol and triglyceride levels, family history, and renal disease. Neither hepatic disease nor vegetarian diet are risk factors for hypertension.

The client tells the nurse that he is sorry he fell off the roof and broke his leg. The nurse responds by saying, "Oh, you poor thing! I am sorry you fell off the roof and broke your leg, too." What type of response is this? A. Empathetic B. Therapeutic C. Sympathetic D. Supportive

ANS: C Feedback: Sympathy is feeling what a client feels from the viewpoint of the nurse. When the nurse is being sympathetic, the nurse is interpreting the situation as the nurse perceives it. The nurse's response is incorrect to the situation. The nurse should be supportive, therapeutic, and empathetic in response to the client's situation.

A client has been prescribed tetracycline for acne. What is the most important point the nurse should make in client teaching about this medication? A. The client may experience phototoxicity B. The medication may interfere with the menstrual cycle C. The client may experience photosensitivity D. The medication may be inactivated by antacids

ANS: C Feedback: Tetracycline can lead to photosensitivity reactions, which usually present as a rash after exposure to the sun. Other medications stimulate phototoxicity, a reaction caused by the drug's molecules absorbing energy from a particular UV wavelength and then damaging surrounding tissues. The result is marked and severely tender sunburn. Tetracycline does not interfere with the menstrual cycle and is not inactivated by antacids.

A nurse is providing client teaching to the parents of a preschooler who experiences chronic epistaxis. What would the nurse identify as the area where most nosebleeds originate? A. Rosenmuller's fossa B. Wharton's ducts C. Kiesselbach's plexus D. Stensen's duct

ANS: C Feedback: The anterior portion of the nasal septum has a rich vascular supply known as Kiesselbach's plexus. It is often the site of origination for nosebleeds. The other options are distracters for the question.

What term refers to the drive to establish and guide the next generation? A. Ego integrity B. Despair C. Generativity D. Stagnation

ANS: C Feedback: The essence of generativity is that people need to be needed. With maturity comes satisfaction from what has been produced. Generativity is principally the drive to establish and guide the next generation. The three distractors are also terms from Erikson's Model of Individual Development. In the late adulthood stage ego integrity versus despair is paired. Ego integrity is when one comes to terms with one's life choices. When one does not reach this developmental mark, despair can result. In the middle adult stage generativity is paired with stagnation. Stagnation results when one does not meet generativity as described above.

The nurse is gathering a complete history of the client's present illness. The nurse knows that the most appropriate way to begin to gather this information is what? A. Assessing the client's vital signs B. Gathering a complete list of the client's medications C. Asking open-ended questions D. Asking focused questions

ANS: C Feedback: The nurse collects information about the present illness by beginning with open-ended questions and having the client explain symptoms. The most appropriate way to collect data about the present illness is not to assess the client's vital signs, gather a complete list of the client's medications, or ask focused questions.

To make a legal entry into the medical record, the nurse must document what? A. Laboratory tests ordered B. Attending physician C. Time of the assessment D. Nature of the assessment

ANS: C Feedback: The nurse must record normal assessment data, abnormal assessment data, and the time of the assessment. The nurse does not have to document laboratory tests ordered, the attending physician, or the nature of the assessment.

When dealing with a client who has impaired hearing, what interventions should the nurse implement to facilitate lip reading? A. Speak louder than usual B. Exaggerate your lip movement C. Sit closer than normally to the client D. Speak much more slowly than usual

ANS: C Feedback: The nurse sits closer to the client with hearing impairment to facilitate a setting for lip reading. The nurse uses regular speech volume and lip movement but may speak slightly more slowly.

When caring for clients in any health care environment, what is the most important technique for preventing infection? A. Sterile technique B. Standard precautions C. Hand hygiene D. Use of gloves

ANS: C Feedback: The single most important action to prevent infection is hand hygiene. Sterile technique is important in preventing infection in invasive procedures, but the questions does not address the specific situation of invasive procedures, only infection in general. Standard precautions and the use of gloves prevent a healthcare provider from being exposed to blood and body fluid while caring for a client. This is important in the infection cycle, but is not the most important technique.

The instructor asks the student why percussion is performed on a client. What would be the student's most accurate response? A. To identify resonance B. To hear bowel sounds C. To elicit tenderness D. To produce tympany

ANS: C Feedback: The third technique of physical assessment is percussion to produce sound or elicit tenderness. Percussion produces sounds that can be described using resonance and tympany, but that is not the most accurate response the student can give. Percussion is not used to hear bowel sounds.

In some countries, the transition to adulthood is a long process. Why is this? A. These countries value parenting more than in other countries B. These countries have no rites of passage C. These countries have higher education opportunities D. These countries do not value independence

ANS: C Feedback: The transition to adulthood is a long process for some cultures (particularly those with higher education opportunities) and a very short one for others. The process is not related to how much a country values parenting or embraces rites of passage. Countries that value independence would be equated with shortened, not prolonged, transitions to adulthood.

The nursing instructor is explaining SBAR documentation to students before taking them into the clinical area. What would the instructor explain that the situation, background, and assessment are based on in SBAR charting? A. The client's background B. Information that the nurse obtains from the family C. Complete and accurate assessment data D. Data in old medical records

ANS: C Feedback: This model organizes communication around Situation, Background, Assessment, and Recommendations. Note that situation, background, and assessment are all based on the collection of complete and accurate assessment data. Situation, background, and assessment are not based entirely on the client's background, what family has to say, or what is in the old medical records.

Which of the following tests would be most appropriate for the nurse to use when assessing motor function of the trigeminal nerve? A. Ask the client to differentiate sharp and dull sensations on the face B. Have the client smile, frown, and wrinkle the forehead C. Palpate the temporal and masseter muscles while the client clenches teeth D. Assess dilatation of pupils with direct light

ANS: C Feedback: To test the motor function of the trigeminal nerve (CN V), the nurse would ask the client to clench the teeth and palpate the temporal and masseter muscles for contraction. Touching the client's face for dullness or sharp sensations tests the sensory function of the trigeminal nerve. Having the client frown, smile, and wrinkle the forehead tests the motor function of the facial nerve (CN VI). Assessing pupillary dilation tests the oculomotor (CN III) nerve.

Pain is often untreated or undertreated; when this occurs, chronic pain may result. What can be the outcome? A. Chronic sensitization syndrome B. Peripheral sensitization syndrome C. Complex regional pain syndrome D. Fibromyalgia pain syndrome

ANS: C Feedback: Untreated or undertreated acute pain may lead to chronic pain syndromes, such as complex regional pain syndrome (CRPS), which are difficult to treat. Chronic sensitization is an excitatory process involving spinal nerves produced by continued pain stimuli that can persist even after peripheral stimulation is no longer present. Peripheral sensitization is the result of an inflammatory process that creates hypersensitivity to touch or pressure. Fibromyalgia pain syndrome is a complex disorder characterized by widespread musculoskeletal pain, as well as various other disorders that could possibly include mood, sleep, and fatigue issues.

A first-time mother calls the clinic to talk to the nurse. The mother is very upset, saying that her newborn's fingernails dip in the middle, appearing spoon-like. What would be the nurse's best response? A. "Take the newborn to the emergency room to be evaluated." B. "Bring your newborn to the clinic immediately." C. "This may be normal in newborns." D. "This is a sign of a nutritional deficiency. What are you feeding your newborn?"

ANS: C Feedback: With koilonychia (spoon nails), the nails may have transverse and longitudinal concavity, with the appearance of a spoon. This sign may be normal in newborns and usually resolves in few months. The nurse would not tell the mother to take the newborn to the emergency room or clinic. The nurse would not tell the mother that the finding was a sign of a nutritional deficiency.

What is the importance of assessing vital signs? (Select all that apply.) A. To carry out orders from the healthcare provider B. To plan how to improve a client's condition C. To establish a baseline D. To monitor risks for alterations in health E. To evaluate the client's responses to treatment

ANS: C, D, E Feedback: Assessment of vital signs helps nurses to establish a baseline, monitor a client's condition, evaluate responses to treatment, identify problems, and monitor risks for alterations in health. Vital signs are not assessed to carry out orders from the healthcare provider or to plan how to improve a client's condition.

The autonomic nervous system innervates the salivary glands along with which cranial nerves? (Select all that apply.) A. X B. VIII C. VII D. IX E. XII

ANS: C, D, E Feedback: The autonomic nervous system and cranial nerves VII (facial) and IX (glossopharyngeal) innervate secretions of the major and minor salivary ducts. Cranial nerve XII (hypoglossal) innervates the submandibular glands. Cranial nerves VIII and X are not involved.

Nursing students are learning about assessment of the head and neck. What cultural considerations would the students learn to assess in relation to this area? (Select all that apply.) A. Shape of the ears B. Shape of the chin C. Shape of the lips D. Shape of the nose E. Shape of the eyes

ANS: C, D, E Feedback: The most noticeable difference among racial groups is skin color. Shape of the eyes, nose, and lips also varies based on background and genetics. Variations in skull or neck shape or size relate more to height and weight than to specific racial or cultural background. Shape of the chin and ears is not related to cultural differences.

The nurse is admitting a client to the clinic and performs a focused assessment. What makes a focused assessment different from a comprehensive assessment? A. A focused assessment covers the body head to toe, unlike a comprehensive assessment B. A focused assessment occurs only in the clinic area, unlike a comprehensive assessment C. A focused assessment involves all body systems, unlike a comprehensive assessment D. A focused assessment is more in-depth on specific issues, unlike a comprehensive assessment

ANS: D Feedback: A focused assessment is based on the client's issues. This type of assessment can occur in all settings, including the clinic, hospital, and home health. It usually involves one or two body systems and is smaller in scope than the comprehensive assessment but is more in-depth on the specific issue(s). The comprehensive assessment includes a head-to-toe evaluation.

When using Gordon's framework for a functional health assessment, the nurse asks a client, "Have you made any changes in your environment because of vision, hearing, or memory decrease?" What functional health pattern is the nurse assessing? A. Vision B. Hearing C. Coping D. Cognition

ANS: D Feedback: A question to include in review of cognition and perception is whether the client has made any environmental changes because of vision, hearing, or memory decrease. The options of vision or hearing individually would not be complete as a response. The option of coping is not addressed in the question posed by the nurse.

When administering a mini-Cog examination (MMSE), what score would indicate cognitive impairment? A. 38 or less B. 33 or less C. 28 or less D. 23 or less

ANS: D Feedback: A score of 23 or lower on the MMSE indicates cognitive impairment. The other options represent scores too high to indicate cognitive impairment.

A mother brings her 6-month-old child to the clinic for a follow-up assessment after antibiotic treatment for recurring otitis media. What would the nurse expect to find while assessing the client's mouth to best confirm a complication of the treatment? A. Swollen uvula B. Red and white patches in the throat C. White patches firmly attached to the buccal mucosa D. White coated tongue

ANS: D Feedback: A white coating of the tongue may be oral candidiasis. This condition is very common in clients taking antibiotics. Red and white patches in the throat might indicate strep throat, which would be an unexpected finding. The uvula may be swollen with allergic reactions. White patches attached to the buccal mucosa suggest leukoplakia.

The nurse can best practice effective care by exhibiting which behavior during a cultural assessment? A. Set a focused time limit for collecting data. B. Stay focused on the computer screen to remain neutral. C. De-emphasize nonverbal communication cues. D. Acknowledge own prejudices that might create barriers to care.

ANS: D Feedback: Acknowledge that the nurse's own beliefs and prejudices might create barriers to providing culturally sensitive care. Leininger suggests that the attributes and behaviors of a nurse practicing effective care within the client's cultural context include the following: - Genuine interest in a client's culture and personal life experiences - Active listening and awareness of meanings behind the client's verbal communication (storytelling) - Nonverbal communication (body language, eye contact, facial expressions, interpersonal space, and preferences regarding touch) - Acknowledgement that the nurse's own beliefs and prejudices might create barriers to providing culturally sensitive care.

During the admission process, the client states, "I am allergic to sulfa drugs." How would the nurse verify this information? A. Ask family members B. Ask the physician C. Ask the client about the response to the allergen D. Compare against the client's medical records

ANS: D Feedback: Allergies are verified with clients and compared against their medical records. When asking about allergies, the nurse notes the type of response such as rash, throat swelling, difficulty breathing, or anaphylactic shock. It is not appropriate to ask either family members or the physician about allergies that the client may have. Asking the client about reactions to an allergen is part of the assessment; it is not to verify the allergy information.

The implementation of computerized charting systems is a nationwide event. What has research shown about the use of computerized systems? A. Safety among client populations decreases B. Pharmacy orders are electronically verified C. Physician notes are more secure D. Client safety increases

ANS: D Feedback: Although implementing a computerized system is expensive and requires much planning and education, such systems significantly increase client safety. Computerized medical records do not decrease client safety, make physician notes more secure, or verify pharmacy orders.

A shared, learned, and symbolic system of values, beliefs, and attitudes that shape and influence how people see and behave in the world is a definition of what term?A. society B. community C. system D. culture

ANS: D Feedback: At the most basic level, culture can be defined as a shared, learned, and symbolic system of values, beliefs, and attitudes that shape and influence how people see and behave in the world. Society is defined as a group of people bound by a common culture. Community is defined as a group of people having a common interest. System is defined as a group of interrelated elements forming a complex whole.

A nurse is preparing to assess a client's cerebellar function. What aspect of neurological function should the nurse address? A. Remote memory B. Sensation C. Mental status exam D. Balance

ANS: D Feedback: Balance and coordination are functions of the pyramidal and extrapyramidal tracts of the motor and cerebellar systems. Remote memory and mental status exam provide information about the client's cognitive ability. Testing for sensation would address issues with specific cranial nerves or problems involving the parietal lobe.

A nurse is teaching a class on hypertension in a community setting. What risk factor would the nurse be sure to address to the class? A. Quitting cigarette smoking 5 years ago B. Loss of 50 pounds within the last 12 months C. High cholesterol and low triglyceride levels D. Family history

ANS: D Feedback: Clients should be educated about the risks of hypertension. Risk factors include obesity, cigarette smoking, heavy alcohol consumption, prolonged stress, high cholesterol and triglyceride levels, family history, and renal disease. Weight loss, low triglyceride level, and smoking cessation are not risk factors for hypertension.

The nursing instructor talks with the student nurse on the adult psychiatric unit. The student tells the instructor that cradle cap appears to be around a specific client's face. The instructor explains that this may be an indication of long-term lack of care because of what disorder? A. Depression B. Bipolar disorder C. Violence D. Schizophrenia

ANS: D Feedback: Cradle cap around the face of adults indicates long-term lack of care and is often seen in clients with schizophrenia. None of the other options are associated with this assessment finding.

A nurse is presenting an educational event for a local community group. What form of knowing would the nurse tell the group is learned and influenced by the individual person's culture? A. Pragmatic mechanics B. Cognitive mechanics C. Fluid intelligence D. Crystallized intelligence

ANS: D Feedback: Crystallized intelligence is "accumulated knowledge and skills based on education and life experiences." Fluid intelligence, also known as cognitive mechanics, looks at the ability for the brain to learn new skills or tasks. Pragmatic mechanics is not a term used to describe this type of knowledge.

An older adult client is admitted to the hospital after a fall during which the client's head was injured. While performing the admission assessment, the nurse finds a large ecchymosis over the C7-T1 area. The client reports tenderness on palpation and movement. What would be an appropriate nursing diagnosis for this client? A. Impaired skin integrity related to bed rest B. Impaired range of motion related to injury C. Immobility related to bed rest D. Impaired comfort related to possible neck injury

ANS: D Feedback: Diagnosis of a cervical spine injury is challenging and, in many cases, goes undiagnosed, especially in those lacking adequate health insurance. Clients at risk include those following a fall or collision and those with osteoporosis, advanced arthritis, cancer, or degenerative bone disease. The scenario does not indicate that the client is on bed rest or that the client has a limited range of motion.

When an agency has policies that require nurses to write notes using the DAR (Data, Action, Response) system, the nursing documentation can include what? A. Areas of personal accomplishments B. Areas of personal weakness C. Social networks D. Family concerns

ANS: D Feedback: Documentation can focus on areas of strengths as well as medical problems, family concerns, or nursing diagnoses. Focus notes do not generally include areas of personal accomplishments, personal weaknesses, or the client's social networks.

When conducting a generalized assessment of a new client, what would the nurse focus upon when inspecting the neck? A. Signs of a strain B. Indications of a vertebral injury C. Lymph node enlargement D. Limitations in movement

ANS: D Feedback: During inspection of the neck, the nurse observes for lesions and limitations in movement. The nurse cannot assess strain, vertebral injury, or lymph node enlargement by inspection. The other options are incorrect because they are related to a focused inspection assessment.

General survey of an adult client reveals that the client is wearing eccentric makeup and clothes. For what would the nurse perform further assessments? A. Inadequate finances B. Depression C. Poor hygiene D. Mania

ANS: D Feedback: Eccentric makeup or dress may indicate mania. Previously well-groomed clients who are now disheveled may have depression. Worn or disheveled clothes may indicate inadequate finances, disinterest in appearance, or problems with self-care.

What is egocentrism? A. Internalized sets of actions that permit children to do mentally what they once did physically B. Centering attention on one aspect of a problem and failing to consider other dimensions C. The belief that inanimate objects are capable of action and have lifelike qualities D. The inability to distinguish one's own perspective from another person's

ANS: D Feedback: Egocentrism is the inability to distinguish one's own perspective from another person's. Animism is the belief that inanimate objects are capable of action and have lifelike qualities. Centration is the child centering attention on one aspect of a problem and failing to consider other dimensions. Piaget defined operations as internalized sets of actions that permit children to do mentally what they once did physically.

A middle aged client is admitted to the observation unit with right lower quadrant pain. The client has not kept down any food or drink for 24 hours. The client's temperature is 38.6°C orally (101.5°F). The client describes the pain as "achy with periods of sharp, stabbing sensations." What would be the most appropriate nutritional nursing diagnosis for a client with these assessment data? A. Deficient knowledge related to disease process B. Pain related to an inability to tolerate food C. Potential for malnutrition (deficit) related to an inability to tolerate food D. Fluid volume less than body requirements related to an inability to tolerate fluids

ANS: D Feedback: Fluid volume is generally affected faster than anything else when a client cannot keep fluids down. A knowledge deficit can only occur once a medical diagnosis is established. Pain is not a nutrition-related nursing diagnosis. Malnutrition does not occur during a hospitalization but over a long period.

The nurse is preparing to assess balance in an older adult client. Which test would the nurse plan on possibly omitting from the exam? A. Romberg B. Tandem walking C. Gait D. Hop on one foot

ANS: D Feedback: Hopping on one foot is often impossible for the older adult because of decreased flexibility and strength and may place the client at risk. The nurse needs to ensure the client's safety by standing close by, especially with tandem walking and Romberg's testing because some older clients may have difficulty with maintaining balance. However, these tests would not be omitted. Older clients may have a slow uncertain gait. Testing the client's gate would not be omitted.

A male client who was transferred from intensive care and extubated less than 24 hours ago exhibits drooling and a weak voice. At meal time, what is the nurse's priority action? A. Delegate feeding the client to a nursing assistant. B. Place the client in semi-Fowler's position for feeding. C. Call the healthcare provider to request a liquid diet. D. Explain to the client why he should not eat anything by mouth yet.

ANS: D Feedback: If the client exhibits drooling with a weak voice and has a history of recent intubation he should be kept on nothing by mouth and a swallowing evaluation should be performed before any type of oral feeding. The client should also be placed on aspiration precautions and remain in a fully upright position.

The nurse practitioner auscultates both lobes of a client's enlarged thyroid gland. Identification of what sound would tend to confirm a diagnosis of a toxic goiter? A. Rush B. Gurgle C. Murmur D. Bruit

ANS: D Feedback: If the thyroid is enlarged, either unilaterally or bilaterally, the nurse uses the bell of the stethoscope to auscultate over each lobe for a bruit. Bruits are most often found with a toxic goiter, hyperthyroidism, or thyrotoxicosis. A murmur is assessed during a cardiac assessment.

A nurse caring for a client admitted 2 days ago following a cerebral vascular accident. The nurse notes that the client is frequently coughing, has food falling from the mouth while eating, and frequently chokes. What would be the most pertinent nursing diagnosis for this client? A. Impaired dentition B. Alteration in mobility C. Altered cerebral perfusion D. Impaired swallowing

ANS: D Feedback: Impaired swallowing is associated with problems in oral, pharyngeal, or esophageal structure or function. Related findings include delayed swallowing; gurgly voice; frequent coughing, choking, or gagging; inability to clear oral cavity; and food falling from the mouth. The scenario described does not mention impaired dentition, alteration in mobility, or altered cerebral perfusion.

While beginning the assessment of a client's abdomen, the nurse starts in the middle of the abdomen and expects to hear high-frequency sounds. What part of the stethoscope will provide the best sound with firm skin contact? A. The bell B. The small side of the chestpiece C. The earpieces D. The diaphragm

ANS: D Feedback: Most stethoscopes have a diaphragm and bell on the chestpiece. The bell is used with light skin contact to hear low-frequency sounds, while the diaphragm is used with firm skin contact to hear high-frequency sounds. The earpieces do not provide sound, and the small side of the chestpiece is the bell.

When assessing cranial nerves IX and X, which of the following would the nurse consider as a normal finding? A. Stationary soft palate on phonation B. Deviation of uvula when client says "ah" C. Asymmetrical soft palate D. Uvula and soft palate rising bilaterally

ANS: D Feedback: Normal findings associated with testing cranial never IX and cranial nerve X include a uvula and soft palate rising bilaterally and symmetrically on phonation. A stationary or asymmetrical soft palate or deviation of the uvula would be considered an abnormal finding.

A nurse is writing a care plan for a newly admitted client. When formulating the diagnostic statements in the care plan, what would the nurse use? A. Rationale B. American Nurses Association recommendations C. Physical assessment skills D. Diagnostic reasoning

ANS: D Feedback: Nurses use diagnostic reasoning and critical thinking to formulate diagnostic statements. Rationale, ANA recommendations, and physical assessment skills are not part of formulating diagnostic statements. Rationale supports the nursing interventions of the nursing care plan. The American Nurses Association does not have recommendations regarding formulation of diagnostic statements for the care plan. Physical assessment skills are important in the assessment step of the nursing process, not the formulation of the diagnostic statements.

What open-ended question might be helpful when assessing abuse with a client demonstrating discomfort discussing the issue? A. "What can I tell you about abuse?" B. "I know you have been abused. Please tell me about it." C. "Abuse is more prevalent than most people think." D. "What would you like me to know?"

ANS: D Feedback: Open-ended questions, such as "What would you like to know?" "How can I help you understand?" or "What would you like me to know?" are especially helpful if a client appears uncomfortable. It would be inappropriate for the nurse to ask what she can tell the client about abuse or to tell the client that the nurse knows the client has been abused. While discussing the prevalence of abuse might help increase the client's knowledge, it might not make her feel any more comfortable about the situation or ready to share information.

Which of the following would lead the nurse to suspect meningeal irritation? A. Hips and knees remain relaxed and motionless when neck is flexed B. Reports of decreased pain with flexion of the hips and knees C. Discomfort behind the knee with full extension of the leg D. Pain and flexion of the hips and knees with neck flexion

ANS: D Feedback: Pain and flexion of the hips and knees are positive Brudzinski's signs and suggest meningeal irritation. Pain and increased resistance to extending the knee when the client's leg is flexed at both the hip and the knee and then straightened are positive Kernig's signs suggesting meningeal irritation. Discomfort behind the knee during full extension when testing for Kernig's sign occurs in many normal people.

Parents bring a child to the clinic and report a "rash" on her knee. On assessment, the nurse practitioner notes the area to be a reddish-pink lesion covered with silvery scales. What would the nurse practitioner chart? A. Seborrhea B. Contact dermatitis C. Eczema D. Psoriasis

ANS: D Feedback: Psoriasis is characterized by reddish-pink lesions covered with silvery scales. It commonly occurs on extensor surfaces such as the elbows and knees but can appear anywhere on the body. Seborrhea is an inflammatory skin disorder characterized by macular lesions that may be pink, red, or orange-yellow and may or may not have a fine scale. Distribution is usually on the face, scalp, and ears. Contact dermatitis is an inflammatory response to an antigen that has contact with exposed skin. Initial contact causes stimulation of the histamine receptors, which results in the classic erythematous and pruritic lesions. Eczema, also known as atopic dermatitis, is characterized by itchy, pink macular or papular lesions, commonly located on flexural areas such as the inner elbows or posterior knees. Eczema can occur anywhere on the body.

While assessing a new client in the clinic area, the nurse administers the SAD PERSONAS. For what is the nurse most likely assessing? A. Risk of depression B. Risk of self-mutilation C. Risk of violence D. Risk of suicide

ANS: D Feedback: SAD PERSONAS suicide risk assessment: Sex, Age, Depression, Previous attempt, Ethanol abuse, Rational thought loss, Social supports lacking, Organized plan, No spouse, Access to lethal means, Sickness. The presence of each factor is given a point value of one. Total scores range from 0 to 10. Higher scores indicate greater client suicide risk. The assessment tool does not assess depression, self-mutilation, or violence.

A nursing student is helping with a group presentation on social assessment. What would be a foundational concept for the student to include in the group presentation? A. Social assessment emphasizes the interconnectedness of physical, physiologic, and educational dimensions of health. B. Social assessment emphasizes the interconnectedness of physical, family, and social dimensions of health. C. Social assessment emphasizes the interconnectedness of physical, spiritual, and psychic dimensions of health. D. Social assessment emphasizes the interconnectedness of physical, psychosocial, and spiritual dimensions of health.

ANS: D Feedback: Social assessment, integral to quality nursing care at every level, emphasizes the interconnectedness of physical, psychosocial, and spiritual dimensions of health for individuals, communities, and populations studied. Psychic, social, and educational dimensions are not dimensions of health emphasized in the social assessment.

Matters of the human soul are referred to as what? A. culture B. ethnicity C. values and beliefs D. spirituality

ANS: D Feedback: Spirituality, in the most fundamental sense, pertains to matters of the human soul, be it a state of mind, a state of being in the world, a journey of self-discovery, or a place outside the five senses. Culture can be defined as a shared, learned, and symbolic system of values, beliefs, and attitudes that shapes and influences the way people see and behave in the world. Ethnicity is one's self-defined race. Values and beliefs are those things that a person or culture sees as having worth.

The nurse is caring for four clients on the short-stay unit. Which client would cause the nurse greatest concern? A. A 10-year-old client with a BP of 103/62 B. A 7-year-old client with a sinus dysrhythmia C. A 77-year-old client with a resting heart rate of 69 beats/min D. An 82-year-old client with a temperature of 37.2°C (99°F)

ANS: D Feedback: Temperatures considered normal for younger adults may constitute fever in older adults. The three other options represent normal findings for the clients described.

A client with a cervical spine injury reports chronic pain. What would be the most appropriate initial nursing intervention for this client? A. Work with medical team to evaluate possible surgery. B. Discuss pharmacologic interventions. C. Educate the client regarding cervical spine pain. D. Assess the client regarding characteristics of the pain.

ANS: D Feedback: The first step would be for the nurse to assess characteristics of the pain. Surgery or pharmacologic interventions would be considered by the whole health care team after more information was gathered. While education is an appropriate intervention, it would not be addressed initially but rather after pain management interventions were implemented.

The nurse is assessing a teenage girl newly admitted to the pediatric unit. The nurse knows that an efficient assessment framework that provides additional modesty for the client is what? A. Body systems B. Functional C. Focused D. Head to toe

ANS: D Feedback: The head-to-toe method is efficient and provides more modesty for clients. The body systems and functional assessment does not address the modesty issue in the question. The focused assessment is not appropriate for the newly admitted client.

What tool does the nurse use to auscultate the client's abdomen? A. None B. Fetoscope C. Otoscope D. Stethoscope

ANS: D Feedback: The nurse uses a stethoscope to perform auscultation, in which movements of air or fluid are heard in the body over the lungs and abdomen. A fetoscope is used to hear the fetal heartbeat. An otoscope is used to view portions of the ear.

A pediatric nurse is doing the initial shift assessments on assigned clients. One of the clients is a toddler with pneumonia. How would the nurse assess this client's skin turgor? A. Pinch a fold of skin on the client's abdomen. B. Pinch a fold of skin on the client's cheek. C. Pinch a fold of skin on the client's upper thigh. D. Pinch a fold of skin on the client's forearm.

ANS: D Feedback: To assess skin turgor in a toddler, the nurse would gently grasp a fold of the client's skin between the fingers and pull up. Then, the nurse would release the fold of skin. This is easiest performed on the dorsal surface of the client's hand or lower arm. The most accurate reflection of turgor in the adult is on the anterior chest, just below the midclavicular area. The nurse would not assess for skin turgor on a fold of skin on the client's abdomen, cheek, or upper thigh.

Both the peripheral and the central nervous systems are involved in the transmission of a pain stimulus. When there is continued input from the peripheral nervous system, what can develop? A. Fibromyalgia B. A peripherally mediated pain syndrome C. Neuronal plasticity D. A centrally mediated pain syndrome

ANS: D Feedback: Transmission of a pain stimulus uses two separate but continuous systems: the peripheral nervous system and the central nervous system. Continued input from the peripheral nervous system can create a centrally mediated pain syndrome, in which pain occurs without a pain stimulus. Fibromyalgia is chronic pain syndrome. Neuronal plasticity is the term for changes in the transmission of the pain stimulus.

A client who only speaks Spanish is admitted to the unit. The client's sister, who speaks English, is in the room when the English-speaking nurse starts the admission assessment. Why would it be inappropriate to use the sister as an interpreter for this client? A. The sister may not tell the client exactly what the nurse says B. The client's sister may not understand medical terminology C. The sister may not be there every time the nurse needs to talk to the client D. The client may not want the sister to know their private information

ANS: D Feedback: Using children in the family, other relatives, or close friends as interpreters violates privacy laws, because clients may not want to share personal information with others. HIPAA guidelines address privacy issues such as this scenario. Even when the client gives permission for the family member to be present, an official interpreter should be present per facility policy. The other options could be true in some situations, but the priority answer addresses privacy, both the client's right to privacy, and the facility's handling of private information.

A nurse in the emergency department is caring for a nonverbal client. What would be the best way for the nurse to assess this client's level of pain? A. Ask the client to draw a picture of the pain B. Ask the paramedics what they think is the client's pain level C. Ask the client to describe the pain D. Ask the family if they have noticed any changes in the client's behavior

ANS: D Feedback: When attempting to perform a pain assessment on a client who cannot self-report pain, the nurse should try to identify any potential causes for pain, observe client behaviors, ask the family or other caregivers if they have noticed any changes in the client's behavior, and attempt an analgesic trial. A nonverbal client cannot describe the pain in spoken words. Asking the client to draw a picture of pain while in the emergency department would not be the best way to assess the client's level of pain, nor would asking paramedics what they think about the client's pain.

The nurse aide reports to the nurse that an older adult client has abnormal vital signs. What is important to remember in this type of situation? A. At client's age, abnormal vital signs are an indication of something serious B. Normal readings get lower with advanced age C. Normal readings get higher with advanced age D. Normal readings vary according to age

ANS: D Feedback: When encountering an abnormal value, the nurse should obtain the vital sign(s) again to assess accuracy. The nurse should also consider whether the client appears to be in distress, noting skin color, respiratory effort, and behavior. Normal readings vary according to age.

The nurse is conducting a physical assessment. The data the nurse would collect vary depending on what? A. How much time the nurse has B. The seriousness of a client's condition C. The client's cooperation D. Onset of current symptoms

B. Feedback: Data that nurses collect during a physical assessment vary depending on a client's acuity (condition), health history, and current symptoms. The data collected during a physical assessment do not depend on how much time the nurse has, how cooperative the client is, or the onset of the current symptoms.

The purpose of a health assessment includes what? (Select all that apply.) A. Identifying the client's major disease process B. Collecting information about the health status of the client C. Clarifying the client's ability to pay for health care D. Evaluating client outcomes E. Synthesizing collected data

B., D., E. Feedback: Health assessment is "gathering information about the health status of the client, analyzing and synthesizing those data, making judgments about nursing interventions based on the findings and evaluating client care outcomes" (AACN, 2008). While the nurse may elicit financial information and information about disease processes during a health assessment, the purposes of the activity are not to identify the client's major disease process or ability to pay.

What is one of the broad goals within nursing? A. To provide cost effective care B. To form broad nursing diagnoses C. To promote self-care D. To treat human responses

D. Feedback: Four broad goals are within nursing: (1) to promote health (state of optimal functioning or well-being with physical, social, and mental components); (2) to prevent illness; (3) to treat human responses to health or illness; and (4) to advocate for individuals, families, communities, and populations. The other options listed are not broad goals. Nursing, focuses on promoting health; while cost-effective care is strived for, is not a part of the broad goal, therefore, this is not a broad goal within nursing. Nursing looks to develop specific nursing diagnoses, not broad. Promoting self-care is important, but does not correctly answer the question.


Kaugnay na mga set ng pag-aaral

Chapter 48: Diabetes MellitusThe nurse is assessing a 55-yr-old female patient with type 2 diabetes who has a body mass index (BMI) of 31 kg/m2 .Which goal in the plan of care is most important for this patient? a. The patient will reach a glycosylated he

View Set

finanacing higher education everfi module 6

View Set

Chapter 3 - The U.S. Court System

View Set

Physical Science Study Guide Chapters 20,21,22

View Set

Econ Practice Mid-term/Mid-Term study set

View Set

Catcher in the Rye, Catcher in the Rye test Questions

View Set

A Midsummer Night's Dream Quotes

View Set

Musculoskeletal imaging, breast, and superficial structures

View Set

Qasid - 1-1 - تجميد عضوية أسامة العجارمة

View Set